Acc 303 midterm exam

Page 1

ACC 303 Week 5 Midterm Exam – Strayer NEW Click On The Link Below to Purchase A+ Graded Material Instant Download http://www.hwgala.com/ACC-303-Week-5-Midterm-Exam-Strayer-NEW-ACC303W5E.htm Midterm Exam: Chapters 1 Through 4 Chapter 1 FINANCIAL ACCOUNTING AND ACCOUNTING STANDARDS IFRS questions are available at the end of this chapter. TRUE-FALSE—Conceptual 1. Financial accounting is the process of identifying, measuring, analyzing, and communicating financial information needed by management to plan, evaluate, and control a company’s operations. 2. Financial statements are the principal means through which a company communicates its financial information to those outside it. 3. Users of financial reports provided by a company use that information to make their capital allocation decisions. 4. An effective process of capital allocation promotes productivity and provides an efficient market for buying and selling securities and obtaining and granting credit. 5. The objective of financial reporting is to provide financial information about the reporting entity that is useful to present and potential equity investors, but not to users who are not investors. 6. Investors are interested in financial reporting because it provides information that is useful for making decisions (decision-usefulness approach). 7. Users of financial accounting statements have both coinciding and conflicting needs for information of various types. 8. The Securities and Exchange Commission appointed the Committee on Accounting Procedure. 9. The passage of a new FASB Standards Statement requires the support of five of the seven board members. 10. Financial Accounting Concepts set forth fundamental objectives and concepts that are used in developing future standards of financial accounting and reporting.


1-2

Test Bank for Intermediate Accounting, Fourteenth Edition

11. The AICPA created the Accounting Principles Board in 1959. 12. The FASB’s Codification integrates existing GAAP, and creates new GAAP. 13. The AICPA’s Code of Professional Conduct requires that members prepare financial statements in accordance with generally accepted accounting principles. 14. GAAP is a product of careful logic or empirical findings and are not influenced by political action. 15. The Public Company Accounting Oversight Board has oversight and enforcement authority and establishes auditing and independence standards and rules. 16. The expectations gap is caused by what the public thinks accountants should do and what accountants think they can do. 17. Financial reports in the early 21st century did not provide any information about a company’s soft assets (intangibles). 18. Accounting standards are now less likely to require the recording or disclosure of fair value information. 19. U.S. companies that list overseas are required to use International Financial Reporting Standards, issued by the International Accounting Standards Board. 20. Ethical issues in financial accounting are governed by the AICPA. True-False Answers—Conceptual

MULTIPLE CHOICE—Conceptual 21.

General-purpose financial statements are the product of a. financial accounting. b. managerial accounting. c. both financial and managerial accounting. d. neither financial nor managerial accounting.

22.

Users of financial reports include all of the following except a. creditors. b. government agencies. c. unions. d. All of these are users.


Financial Accounting and Accounting Standards

1-3

23.

The financial statements most frequently provided include all of the following except the a. balance sheet. b. income statement. c. statement of cash flows. d. statement of retained earnings.

24.

The information provided by financial reporting pertains to a. individual business enterprises, rather than to industries or an economy as a whole or to members of society as consumers. b. business industries, rather than to individual enterprises or an economy as a whole or to members of society as consumers. c. individual business enterprises, industries, and an economy as a whole, rather than to members of society as consumers. d. an economy as a whole and to members of society as consumers, rather than to individual enterprises or industries.

25.

All the following are differences between financial and managerial accounting in how accounting information is used except to a. plan and control company's operations. b. decide whether to invest in the company. c. evaluate borrowing capacity to determine the extent of a loan to grant. d. All the above.

26.

Which of the following represents a form of communication through financial reporting but not through financial statements? a. Balance sheet. b. President's letter. c. Income statement. d. Notes to financial statements.

P

27.

The process of identifying, measuring, analyzing, and communicating financial information needed by management to plan, evaluate, and control an organization’s operations is called a. financial accounting. b. managerial accounting. c. tax accounting. d. auditing.

28.

How does accounting help the capital allocation process attract investment capital? a. Provides timely, relevant information. b. Encourages innovation. c. Promotes productivity. d. a and b above.

29.

Whether a business is successful and thrives is determined by a. markets. b. free enterprise.


1-4

Test Bank for Intermediate Accounting, Fourteenth Edition

c. competition. d. all of these. 30.

An effective capital allocation process a. promotes productivity. b. encourages innovation. c. provides an efficient market for buying and selling securities. d. all of these.

31.

Financial statements in the early 2000s provide information related to a. nonfinancial measurements. b. forward-looking data. c. hard assets (inventory and plant assets). d. none of these.

32.

Which of the following is not a major challenge facing the accounting profession? a. Nonfinancial measurements. b. Timeliness. c. Accounting for hard assets. d. Forward-looking information.

33.

What is the objective of financial reporting? a. Provide information that is useful to management in making decisions. b. Provide information that clearly portray nonfinancial transactions. c. Provide information about the reporting entity that is useful to present and potential equity investors, lenders, and other creditors. d. Provide information that excludes claims to the resources.

34.

Primary users for general-purpose financial statements include a. creditors. b. employees. c. investors. d. both creditors and investors.

35.

When making decisions, investors are interested in assessing a. the company’s ability to generate net cash inflows. b. management’s ability to protect and enhance the capital providers’ investments. c. Both a and b. d. the company’s ability to generate net income.

36.

Accrual accounting is used because a. cash flows are considered less important. b. it provides a better indication of ability to generate cash flows than the cash basis. c. it recognizes revenues when cash is received and expenses when cash is paid. d. none of the above.


Financial Accounting and Accounting Standards

1-5

37.

Which perspective is adopted as part of the objective of general-purpose financial reporting? a. Decision-usefulness perspective. b. Proprietary perspective. c. Entity perspective. d. Financial reporting perspective.

38.

Accounting principles are "generally accepted" only when a. an authoritative accounting rule-making body has established it in an official pronouncement. b. it has been accepted as appropriate because of its universal application. c. both a and b. d. neither a nor b.

39.

A common set of accounting standards and procedures are called a. financial accounting standards. b. generally accepted accounting principles. c. objectives of financial reporting. d. statements of financial accounting concepts.


1-6

Test Bank for Intermediate Accounting, Fourteenth Edition

40.

Which of the following is a general limitation of "general purpose financial statements"? a. General purpose financial statements may not be the most informative for a specific enterprise. b. General purpose financial statements are comparable. c. General purpose financial statements are assumed to present fairly the company's financial operations. d. None of the above.

41.

What is the relationship between the Securities and Exchange Commission and accounting standard setting in the United States? a. The SEC requires all companies listed on an exchange to submit their financial statements to the SEC. b. The SEC coordinates with the AICPA in establishing accounting standards. c. The SEC has a mandate to establish accounting standards for enterprises under its jurisdiction. d. The SEC reviews financial statements for compliance.

42.

What is due process in the context of standard setting at the FASB? a. FASB operates in full view of the public. b. Public hearings are held on proposed accounting standards. c. Interested parties can make their views known. d. All of the above.

43.

Which of the following organizations has been responsible for setting U.S. accounting standards? a. Accounting Principles Board. b. Committee on Accounting Procedure. c. Financial Accounting Standards Board. d. All of the above.

44.

Why did the AICPA create the Accounting Principles Board? a. The SEC disbanded the previous standard setting organization. b. The previous standard setting organization did not provide a structured set of accounting principles. c. No such organization existed in the past. d. None of the above.

45.

Which organization was responsible for issuing Accounting Research Bulletins? a. Accounting Principles Board. b. Committee on Accounting Procedure. c. The SEC. d. AICPA.

46.

A characteristic of generally accepted accounting principles include the following: a. common set of standards and principles. b. standards and principles are based federal statutes. c. acceptance requires an affirmative vote of Certified Public Accountants.


Financial Accounting and Accounting Standards

d. practices that become accepted for at least a year by all industry members.

1-7


1-8

Test Bank for Intermediate Accounting, Fourteenth Edition

47.

Characteristics of generally accepted accounting principles include all of the following except a. authoritative accounting the rule-making body established a principle of reporting. b. standards are considered useful by the profession. c. each principle is approved by the SEC. d. practice has become universally accepted over time.

48.

Why was it believed that accounting standards that were issued by the Financial Accounting Standards Board would carry more weight? a. Smaller membership. b. FASB board members are well-paid. c. FASB board members must be CPAs. d. Due process.

49.

The passage of a new FASB Standards Statement requires the support of a. all Board members. b. three Board members. c. four Board members. d. five Board members.

50.

What is the purpose of Emerging Issues Task Force? a. Provide interpretation of existing standards. b. Provide a consensus on how to account for new and unusual financial transactions. c. Provide interpretive guidance. d. Provide timely guidance on select issues.

51.

Which organization is responsible for issuing Emerging Issues Task Force Statements? a. FASB b. CAP c. APB d. SEC

52.

The role of the Securities and Exchange Commission in the formulation of accounting principles can be best described as a. consistently primary. b. consistently secondary. c. sometimes primary and sometimes secondary. d. non-existent.

53.

The body that has the power to prescribe the accounting practices and standards to be employed by companies that fall under its jurisdiction is the a. FASB. b. AICPA. c. SEC. d. APB.


Financial Accounting and Accounting Standards

1-9

54.

Companies that are listed on a stock exchange are required to submit their financial statements to the a. AICPA. b. APB c. FASB. d. SEC.

55.

The Financial Accounting Standards Board (FASB) was proposed by the a. American Institute of Certified Public Accountants. b. Accounting Principles Board. c. Study Group on the Objectives of Financial Statements. d. Special Study Group on establishment of Accounting Principles (Wheat Committee).

56.

The Financial Accounting Standards Board a. has issued a series of pronouncements entitled Statements on Auditing Standards. b. was the forerunner of the current Accounting Principles Board. c. is the arm of the Securities and Exchange Commission responsible for setting financial accounting standards. d. is appointed by the Financial Accounting Foundation.

57.

The Financial Accounting Foundation a. oversees the operations of the FASB. b. oversees the operations of the AICPA. c. provides information to interested parties on financial reporting issues. d. works with the Financial Accounting Standards Advisory Council to provide information to interested parties on financial reporting issues.

58.

The major distinction between the Financial Accounting Standards Board (FASB) and its predecessor, the Accounting Principles Board (APB), is a. the FASB issues exposure drafts of proposed standards. b. all members of the FASB are fully remunerated, serve full time, and are independent of any companies or institutions. c. all members of the FASB possess extensive experience in financial reporting. d. a majority of the members of the FASB are CPAs drawn from public practice.

59.

The Financial Accounting Standards Board employs a "due process" system which a. is an efficient system for collecting dues from members. b. enables interested parties to express their views on issues under consideration. c. identifies the accounting issues that are the most important. d. requires that all accountants must receive a copy of financial standards.

60.

Which of the following is not a publication of the FASB? a. Statements of Financial Accounting Concepts b. Accounting Research Bulletins c. Interpretations d. Technical Bulletins


1 - 10

Test Bank for Intermediate Accounting, Fourteenth Edition

61.

FASB Technical Bulletins a. are similar to FASB Interpretations in that they establish enforceable standards under the AICPA's Code of Professional Ethics. b. are issued monthly by the FASB to deal with current topics. c. are not expected to have a significant impact on financial reporting in general and provide guidance when it does not conflict with any broad fundamental accounting principle. d. were recently discontinued by the FASB because they dealt with specialized topics having little impact on financial reporting in general.

62.

The purpose of the Emerging Issues Task Force is to a. develop a conceptual framework as a frame of reference for the solution of future problems. b. lobby the FASB on issues that affect a particular industry. c. do research on issues that relate to long-term accounting problems. d. issue statements which reflect a consensus on how to account for new and unusual financial transactions that need to be resolved quickly.

63.

The American Institute of Certified Public Accountants (AICPA) continues to be involved in all of the following except a. developing and enforcing professional ethics. b. developing auditing standards. c. providing professional education programs. d. all of the above.

P

64.

Which of the following pronouncements were issued by the Accounting Principles Board? a. Accounting Research Bulletins b. Opinions c. Statements of Position d. Statements of Financial Accounting Concepts

65.

Which of the following organizations has not been instrumental in the development of financial accounting standards in the United States? a. AICPA b. FASB c. IASB d. SEC

66.

An organization that has not published accounting standards is the a. American Institute of Certified Public Accountants. b. Securities and Exchange Commission. c. Financial Accounting Standards Board. d. All of these have published accounting standards.

67.

The purpose of Statements of Financial Accounting Concepts is to a. establish GAAP. b. modify or extend the existing FASB Standards Statement.


Financial Accounting and Accounting Standards

c. form a conceptual framework for solving existing and emerging problems. d. determine the need for FASB involvement in an emerging issue.

1 - 11


1 - 12

Test Bank for Intermediate Accounting, Fourteenth Edition

P

Members of the Financial Accounting Standards Board are a. employed by the American Institute of Certified Public Accountants (AICPA). b. part-time employees. c. required to hold a CPA certificate. d. independent of any other organization.

P

The following are part of the "due process" system used by the FASB in the evolution of a typical FASB Statement of Financial Accounting Standards:

68.

69.

1. Exposure Draft 2. Statement of Financial Accounting Standards 3. Preliminary Views The chronological order in which these items are released is as follows: a. 1, 2, 3. b. 1, 3, 2. c. 2, 3, 1. d. 3, 1, 2. 70.

Generally accepted accounting principles a. include detailed practices and procedures as well as broad guidelines of general application. b. are influenced by pronouncements of the SEC and IRS. c. change over time as the nature of the business environment changes. d. all of these.

71.

The most significant current source of generally accepted accounting principles is the a. AICPA. b. SEC. c. APB. d. FASB.

72.

Which of the following is not a part of generally accepted accounting principles? a. FASB Interpretations b. CAP Accounting Research Bulletins c. APB Opinions d. All of these are part of generally accepted accounting principles.

73.

Which of the following publications does not qualify as a statement of generally accepted accounting principles? a. Statements of financial standards issued by the FASB b. Accounting interpretations issued by the FASB c. APB Opinions d. Accounting research studies issued by the AICPA

74.

Rule 203 of the Code of Professional Conduct addresses: a. ethical requirements. b. financial statements should be based on generally accepted accounting principles.


Financial Accounting and Accounting Standards

c. advertising to obtained clients. d. auditing financial statements.

1 - 13


1 - 14

Test Bank for Intermediate Accounting, Fourteenth Edition

75.

What is the purpose of a FASB Staff Position? a. Provide interpretation of existing standards. b. Provide a consensus on how to account for new and unusual financial transactions. c. Provide interpretive guidance. d. Provide timely guidance on select issues.

76.

Which of the following is not considered a component of generally accepted accounting principles? a. FASB Implementation Guides. b. Widely recognized industry practices. c. Articles published in CPA journals. d. AICPA Accounting Interpretations.

77.

Financial accounting standard-setting in the United States a. can be described as a social process which reflects political actions of various interested user groups as well as a product of research and logic. b. is based solely on research and empirical findings. c. is a legalistic process based on rules promulgated by governmental agencies. d. is democratic in the sense that a majority of accountants must agree with a standard before it becomes enforceable.

78.

The purpose of the International Accounting Standards Board is to a. issue enforceable standards which regulate the financial accounting and reporting of multinational corporations. b. develop a uniform currency in which the financial transactions of companies throughout the world would be measured. c. promote uniform accounting standards among countries of the world. d. arbitrate accounting disputes between auditors and international companies.

79.

What is not a source of pressure that may influence the accounting standard setting process? a. Congress. b. Lobbyist. c. CPA firms. d. None of the above.

80.

What is a possible danger if politics plays too big a role in accounting standard setting? a. Accounting standards that are not truly generally accepted. b. Individuals may influence the standards. c. User groups become active. d. The FASB delegates its authority to elected officials.


Financial Accounting and Accounting Standards

1 - 15

81.

What is "expectation gap"? a. The difference between what the public thinks the accountant is not doing and what the accountant knows they don't do. b. The difference between what the public thinks the accountant is doing and what Congress says the accountant is doing. c. The difference between what the public thinks the accountant is doing and what the accountant thinks they can do. d. The difference between what the accountant is doing and what the Courts say the accountant should be doing.

82.

What is not a reason that accounting standards may differ across countries? a. Governments. b. Language. c. Culture. d. Past Practice.

83.

What would be an advantage of having all countries adopt and follow the same accounting standards? a. Consistency. b. Comparability. c. Lower preparation costs. d. b and c

84.

Which of the following is an ethical concern of accountants? a. Earnings manipulation. b. Conservative accounting. c. Industry practices. d. None of the above.

Multiple Choice Answers—Conceptual


1 - 16

Test Bank for Intermediate Accounting, Fourteenth Edition

IFRS QUESTIONS True/False: 1. IFRS includes both International Financial Reporting Standards and International Accounting Standards. 2. International Financial Reporting Standards preceded International Accounting Standards 3. The standard-setting structure used by the International Accounting Standards Board is very similar to that used by the Financial Accounting Standards Board. 4. The rules-based standards of IFRS are more detailed than the simpler, principles-based standards of U.S. GAAP. 5. The International Accounting Standards Board issues International Financial Reporting Standards. 6. International Accounting Standards are no longer considered part of IFRS because they have been replaced by International Financial Reporting Standards. Answers to True/False questions:

Multiple Choice: 1. Authoritative standards for IFRS include: a. International Financial Reporting Standards only. b. International Financial Reporting Standards and International Accounting Standards only. c. International Financial Reporting Standards, International Accounting Standards and U.S. GAAP only. d. International Financial Reporting Standards, International Accounting Standards and any GAAP standard recognized by an organized stock exchange.


Financial Accounting and Accounting Standards

1 - 17

2. Which of these statements regarding the IFRS and U.S. GAAP is correct? a. U.S. GAAP is considered to be "principles-based" and more detailed than IFRS. b. U.S. GAAP is considered to be "rules-based" and less detailed than IFRS. c. IFRS is considered to be "principles-based" and less detailed than U.S. GAAP d. Both U.S. GAAP and IFRS are considered to be "rules-based", but U.S. GAAP tends to be more complex. 3. The IASB's standard-setting structure includes all of the following except a. Standing Interpretations Committee b. Standards Advisory Council c. Standards Comparison Committee d. Trustees Answers to Multiple Choice: Short Answer: 1. Why would it be advantageous for U.S. GAAP and International GAAP to be the same? 2. What is the difference between principles-based and rules-based accounting rules? Is IFRS more principles-based than U.S. GAAP? Explain.

CHAPTER 2 CONCEPTUAL FRAMEWORK UNDERLYING FINANCIAL ACCOUNTING IFRS questions are available at the end of this chapter. TRUE-FALSE—Conceptual 1. A soundly developed conceptual framework enables the FASB to issue more useful and consistent pronouncements over time. 2. A conceptual framework is a coherent system of concepts that flow from an objective. 3. The first level of the conceptual framework identifies the recognition, measurement, and disclosure concepts used in establishing accounting standards. 4. The IASB has also issued a conceptual framework and the FASB and the IASB have agreed to develop a common conceptual framework. 5. Although the FASB has developed a conceptual framework, no Statements of Financial Accounting Concepts have been issued to date.


1 - 18

Test Bank for Intermediate Accounting, Fourteenth Edition

6. The objective of financial reporting is the foundation of the conceptual framework. 7. Users of financial statements are assumed to need no knowledge of business and financial accounting matters to understand information contained in financial statements. 8. Relevance and faithful representation are the two primary qualities that make accounting information useful for decision making. 9. The idea of consistency does not mean that companies cannot switch from one accounting method to another. 10. Timeliness and neutrality are two ingredients of relevance. 11. Verifiability and predictive value are two ingredients of faithful representation. 12. Revenues, gains, and distributions to owners all increase equity. 13. Comprehensive income includes all changes in equity during a period except those resulting from investments by owners and distributions to owners. 14. The historical cost principle would be of limited usefulness if not for the going concern assumption. 15. The economic entity assumption means that economic activity can be identified with a particular legal entity. 16. The expense recognition principle states that debits must equal credits in each transaction. 17. Revenues are realizable when assets received or held are readily convertible into cash or claims to cash. 18. Supplementary information may include details or amounts that present a different perspective from that adopted in the financial statements. 19. In order to justify reguiring a particular measurement or disclosure, the benefits to be derived from it must equal the costs associated with it. 20. Prudence or conservatism means when in doubt, choose the solution that will be least likely to overstate liabilities or expenses. True False Answers—Conceptual MULTIPLE CHOICE—Conceptual 21.

Generally accepted accounting principles


Financial Accounting and Accounting Standards

1 - 19

a. are fundamental truths or axioms that can be derived from laws of nature. b. derive their authority from legal court proceedings. c. derive their credibility and authority from general recognition and acceptance by the accounting profession. d. have been specified in detail in the FASB conceptual framework. 22.

A soundly developed conceptual framework of concepts and objectives should a. increase financial statement users' understanding of and confidence in financial reporting. b. enhance comparability among companies' financial statements. c. allow new and emerging practical problems to be more quickly solved. d. all of these.

23.

Which of the following are not true concerning a conceptual framework in account-ing? a. It should be a basis for standard-setting. b. It should allow practical problems to be solved more quickly by reference to it. c. It should be based on fundamental truths that are derived from the laws of nature. d. All of the above (a-c) are true.

24.

What is a purpose of having a conceptual framework? a. To enable the profession to more quickly solve emerging practical problems. b. To provide a foundation from which to build more useful standards. c. Neither a nor b. d. Both a and b.


1 - 20 S

Test Bank for Intermediate Accounting, Fourteenth Edition

25.

Which of the following is not a benefit associated with the FASB Conceptual Framework Project? a. A conceptual framework should increase financial statement users' understanding of and confidence in financial reporting. b. Practical problems should be more quickly solvable by reference to an existing conceptual framework. c. A coherent set of accounting standards and rules should result. d. Business entities will need far less assistance from accountants because the financial reporting process will be quite easy to apply.

26.

In the conceptual framework for financial reporting, what provides "the why"--the purpose of accounting? a. Recognition, measurement, and disclosure concepts such as assumptions, principles, and constraints b. Qualitative characteristics of accounting information c. Elements of financial statements d. Objective of financial reporting

27.

The underlying theme of the conceptual framework is a. decision usefulness. b. understandability. c. faithful representation. d. comparability.

28.

Which of the following is not an objective of financial reporting? a. To provide information about economic resources, the claims to those resources, and the changes in them. b. To provide information that is helpful to investors and creditors and other users in assessing the amounts, timing, and uncertainty of future cash flows. c. To provide information that is useful to those making investment and credit decisions. d. All of these are objectives of financial reporting.

P

29.

The objectives of financial reporting include all of the following except to provide information that a. is useful to the Internal Revenue Service in allocating the tax burden to the business community. b. is useful to those making investment and credit decisions. c. is helpful in assessing future cash flows. d. identifies the economic resources (assets), the claims to those resources (liabilities), and the changes in those resources and claims.

30.

What is a primary objective of financial reporting as indicated in the conceptual framework? a. provide information that is useful to those making investing and credit decisions. b. provide information that is useful to management. c. provide information about those investing in the entity. d. All of the above.


Financial Accounting and Accounting Standards

1 - 21


1 - 22

Test Bank for Intermediate Accounting, Fourteenth Edition

31.

What is a primary objective of financial reporting as indicated in the conceptual framework? a. Provide information that is helpful to present and potential investors, creditors, and other users in assessing the amounts, timing, and uncertainty of future cash flows. b. Provide information that is helpful to present investors, creditors, and other users in assessing the amounts, timing, and uncertainty of future cash flows. c. Provide information that is helpful to potential investors, creditors, and other users in assessing the amounts, timing, and uncertainty of future cash flows. d. None of the above.

32.

Which of the following is a fundamental characteristic of useful accounting information? a. Comparability. b. Relevance. c. Neutrality. d. Materiality.

33.

Which of the following is a primary characteristic of useful accounting information? a. Conservatism. b. Comparability. c. Faithful representation. d. Consistency.

34.

What is meant by comparability when discussing financial accounting information? a. Information has predictive or confirmatory value. b. Information is reasonably free from error. c. Information that is measured and reported in a similar fashion across companies. d. Information is timely.

35.

What is meant by consistency when discussing financial accounting information? a. Information that is measured and reported in a similar fashion across points in time. b. Information is timely. c. Information is measured similarly across the industry. d. Information is verifiable.

36.

Which of the following is an ingredient of relevance? a. Verifiability. b. Neutrality. c. Timeliness. d. Materiality.

37.

Which of the following is an ingredient of faithful representation? a. Predictive value. b. Materiality. c. Neutrality. d. Confirmatory value.


Financial Accounting and Accounting Standards

38.

39.

1 - 23

Changing the method of inventory valuation should be reported in the financial statements under what qualitative characteristic of accounting information? a. Consistency. b. Verifiability. c. Timeliness. d. Comparability. Company A issuing its annual financial reports within one month of the end of the year is an example of which ingredient of fundamental quality of accounting information? a. Neutrality. b. Timeliness. c. Predictive value. d. Completeness.

40.

What is the quality of information that enables users to better forecast future operations? a. Faithful representation. b. Materiality. c. Timeliness. d. Relevance.

41.

Neutrality is an ingredient of which fundamental quality of information? a. Faithful representation. b. Comparability. c. Relevance. d. Understandability.

42.

If the FIFO inventory method was used last period, it should be used for the current and following periods because of a. relevance. b. neutrality. c. understandability. d. consistency.

43.

The pervasive criterion by which accounting information can be judged is that of a. decision usefulness. b. freedom from bias. c. timeliness. d. comparability.

44.

The two fundamental qualities that make accounting information useful for decision making are a. comparability and timeliness. b. materiality and neutrality. c. relevance and faithful representation. d. faithful representation and comparability.

45.

Accounting information is considered to be relevant when it


1 - 24

Test Bank for Intermediate Accounting, Fourteenth Edition

a. can be depended on to represent the economic conditions and events that it is intended to represent. b. is capable of making a difference in a decision. c. is understandable by reasonably informed users of accounting information. d. is verifiable and neutral. 46.

The quality of information that means the numbers and descriptions match what really existed or happened is a. relevance. b. faithful representation. c. completeness. d. neutrality.


Financial Accounting and Accounting Standards

1 - 25

47.

Which of the following does not relate to relevance? a. Materiality b. Predictive value c. Confirmatory value d. All of these

48.

According to Statement of Financial Accounting Concepts No. 2, materiality is an ingredient of the fundamental quality of Relevance Faithful Representation a. Yes Yes b. No Yes c. Yes No d. No No

49.

According to Statement of Financial Accounting Concepts No. 2, completeness is an ingredient of the fundamental quality of Relevance Faithful Representation a. Yes No b. Yes Yes c. No No d. No Yes

50.

According to Statement of Financial Accounting Concepts No. 2, neutrality is an ingredient of the fundamental quality of Relevance Faithful Representation a. Yes Yes b. No Yes c. Yes No d. No No

51.

Neutrality means that information a. provides benefits which are at least equal to the costs of its preparation. b. can be compared with similar information about an enterprise at other points in time. c. would have no impact on a decision maker. d. cannot favor one set of interested parties over another.

52.

The characteristic that is demonstrated when a high degree of consensus can be secured among independent measurers using the same measurement methods is a. relevance. b. faithful representation. c. verifiability. d. neutrality.

53.

According to Statement of Financial Accounting Concepts No. 2, predictive value is an ingredient of the fundamental quality of Relevance Faithful Representation a. Yes No


1 - 26

Test Bank for Intermediate Accounting, Fourteenth Edition

b. Yes Yes c. No No d. No Yes 54.

Under Statement of Financial Accounting Concepts No. 2, free from error is an ingredient of the fundamental quality of Faithful Representation Relevance a. Yes Yes b. No Yes c. Yes No d. No No

55.

Financial information does not demonstrate consistency when a. firms in the same industry use different accounting methods to account for the same type of transaction. b. a company changes its estimate of the salvage value of a fixed asset. c. a company fails to adjust its financial statements for changes in the value of the measuring unit. d. none of these.

56.

Financial information exhibits the characteristic of consistency when a. expenses are reported as charges against revenue in the period in which they are paid. b. companies apply the same accounting treatment to similar events, from period to period. c. extraordinary gains and losses are not included on the income statement. d. accounting procedures are adopted which give a consistent rate of net income.

57.

Information about different companies and about different periods of the same company can be prepared and presented in a similar manner. Comparability and consistency are related to which of these objectives? Comparability Consistency a. companies companies b. companies Periods c. Periods companies d. Periods Periods

58.

When information about two different enterprises has been prepared and presented in a similar manner, the information exhibits the characteristic of a. relevance. b. faithful representation. c. consistency. d. none of these.

59.

The elements of financial statements include investments by owners. These are increases in an entity's net assets resulting from owners' a. transfers of assets to the entity. b. rendering services to the entity.


Financial Accounting and Accounting Standards

1 - 27

c. satisfaction of liabilities of the entity. d. all of these. 60.

61.

In classifying the elements of financial statements, the primary distinction between revenues and gains is a. the materiality of the amounts involved. b. the likelihood that the transactions involved will recur in the future. c. the nature of the activities that gave rise to the transactions involved. d. the costs versus the benefits of the alternative methods of disclosing the transactions involved. A decrease in net assets arising from peripheral or incidental transactions is called a(n) a. capital expenditure. b. cost. c. loss. d. expense.

62.

One of the elements of financial statements is comprehensive income. As described in Statement of Financial Accounting Concepts No. 6, "Elements of Financial Statements," comprehensive income is equal to a. revenues minus expenses plus gains minus losses. b. revenues minus expenses plus gains minus losses plus investments by owners minus distributions to owners. c. revenues minus expenses plus gains minus losses plus investments by owners minus distributions to owners plus assets minus liabilities. d. none of these.

63.

Which of the following elements of financial statements is not a component of comprehensive income? a. Revenues b. Distributions to owners c. Losses d. Expenses

P

Which of the following is false with regard to the element "comprehensive income"? a. It is more inclusive than the traditional notion of net income. b. It includes net income and all other changes in equity exclusive of owners' investments and distributions to owners. c. This concept is not yet being applied in practice. d. It excludes prior period adjustments (transactions that relate to previous periods, such as corrections of errors).

S

According to the FASB conceptual framework, which of the following elements describes transactions or events that affect a company during a period of time? a. Assets. b. Expenses. c. Equity. d. Liabilities.

64.

65.


Test Bank for Intermediate Accounting, Fourteenth Edition

1 - 28 S

66.

According to the FASB Conceptual Framework, the elementsď‚žassets, liabilities, and equityď‚ždescribe amounts of resources and claims to resources at/during a a. b. c. d.

Moment in Time Yes No Yes Yes No Yes No No

Period of Time

67.

Which of the following is not a basic element of financial statements? a. Assets. b. Balance sheet. c. Losses. d. Revenue.

68.

Which of the following basic elements of financial statements is more associated with the balance sheet than the income statement? a. Equity. b. Revenue. c. Gains. d. Expenses.

69.

Issuance of common stock for cash affects which basic element of financial statements? a. Revenues. b. Losses. c. Liabilities. d. Equity.

70.

Which basic element of financial statements arises from peripheral or incidental transactions? a. Assets. b. Liabilities. c. Gains. d. Expenses.

71.

Which of the following is not a basic assumption underlying the financial accounting structure? a. Economic entity assumption. b. Going concern assumption. c. Periodicity assumption. d. Historical cost assumption.

72.

Which basic assumption is illustrated when a firm reports financial results on an annual basis? a. Economic entity assumption. b. Going concern assumption. c. Periodicity assumption.


Financial Accounting and Accounting Standards

1 - 29

d. Monetary unit assumption. 73.

Which basic assumption may not be followed when a firm in bankruptcy reports financial results? a. Economic entity assumption. b. Going concern assumption. c. Periodicity assumption. d. Monetary unit assumption.

74.

Which accounting assumption or principle is being violated if a company provides financial reports in connection with a new product introduction? a. Economic entity. b. Periodicity. c. Revenue recognition. d. Full disclosure.


Test Bank for Intermediate Accounting, Fourteenth Edition

1 - 30 S

Which of the following basic accounting assumptions is threatened by the existence of severe inflation in the economy? a. Monetary unit assumption. b. Periodicity assumption. c. Going-concern assumption. d. Economic entity assumption.

S

During the lifetime of an entity accountants produce financial statements at artificial points in time in accordance with the concept of

75.

76.

a. b. c. d.

Relevance No No Yes No No Yes YesYes

Periodicity

77.

Under current GAAP, inflation is ignored in accounting due to the a. economic entity assumption. b. going concern assumption. c. monetary unit assumption. d. periodicity assumption.

78.

The economic entity assumption a. is inapplicable to unincorporated businesses. b. recognizes the legal aspects of business organizations. c. requires periodic income measurement. d. is applicable to all forms of business organizations.

79.

Preparation of consolidated financial statements when a parent-subsidiary relationship exists is an example of the a. economic entity assumption. b. relevance characteristic. c. comparability characteristic. d. neutrality characteristic.

80.

During the lifetime of an entity, accountants produce financial statements at arbitrary points in time in accordance with which basic accounting concept? a. Cost constraint b. Periodicity assumption c. Conservatism constraint d. Expense recognition principle

81.

What accounting concept justifies the usage of depreciation and amortization policies? a. Going concern assumption b. Fair value principle c. Full disclosure principle d. Monetary unit assumption


Financial Accounting and Accounting Standards

1 - 31

82.

The assumption that a company will not be sold or liquidated in the near future is known as the a. economic entity assumption. b. monetary unit assumption. c. periodicity assumption. d. none of these.

83.

Which of the following is an implication of the going concern assumption? a. The historical cost principle is credible. b. Depreciation and amortization policies are justifiable and appropriate. c. The current-noncurrent classification of assets and liabilities is justifiable and signifycant. d. All of these.

84.

Proponents of historical cost ordinarily maintain that in comparison with all other valuation alternatives for general purpose financial reporting, statements prepared using historical costs are more a. faithfully representative. b. relevant. c. indicative of the entity's purchasing power. d. conservative.

85.

Valuing assets at their liquidation values rather than their cost is inconsistent with the a. periodicity assumption. b. expense recognition principle. c. materiality constraint. d. historical cost principle.

86.

Revenue is generally recognized when realized or realizable and earned. This statement describes the a. consistency characteristic. b. expense recognition principle. c. revenue recognition principle. d. relevance characteristic.

87.

Generally, revenue from sales should be recognized at a point when a. management decides it is appropriate to do so. b. the product is available for sale to the ultimate consumer. c. the entire amount receivable has been collected from the customer and there remains no further warranty liability. d. none of these.

88.

Revenue generally should be recognized a. at the end of production. b. at the time of cash collection. c. when realized. d. when realized or realizable and earned.


1 - 32

Test Bank for Intermediate Accounting, Fourteenth Edition


Financial Accounting and Accounting Standards

1 - 33

89.

Which of the following is not a time when revenue may be recognized? a. At time of sale b. At receipt of cash c. During production d. All of these are possible times of revenue recognition.

90.

Which of the following is the process of converting assets received or held into cash or claims to cash? a. Recognition b. Measurement c. Realization d. Allocation

91.

"When products (goods or services), merchandise, or other assets are exchanged for cash or claims to cash" is a definition of a. allocated. b. realized. c. realizable. d. earned.

92.

The allowance for doubtful accounts, which appears as a deduction from accounts receivable on a balance sheet and which is based on an estimate of bad debts, is an application of the a. consistency characteristic. b. expense recognition principle. c. materiality constraint. d. revenue recognition principle.

93.

The accounting principle of expense recognition is best demonstrated by a. not recognizing any expense unless some revenue is realized. b. associating effort (expense) with accomplishment (revenue). c. recognizing prepaid rent received as revenue. d. establishing an Appropriation for Contingencies account.

94.

Which of the following serves as the justification for the periodic recording of depreciation expense? a. Association of efforts (expense) with accomplishments (revenue) b. Systematic and rational allocation of cost over the periods benefited c. Immediate recognition of an expense d. Minimization of income tax liability

95.

Application of the full disclosure principle a. is theoretically desirable but not practical because the costs of complete disclosure exceed the benefits. b. is violated when important financial information is buried in the notes to the financial statements.


1 - 34

Test Bank for Intermediate Accounting, Fourteenth Edition

c. is demonstrated by the use of supplementary information presenting the effects of changing prices. d. requires that the financial statements be consistent and comparable.


Financial Accounting and Accounting Standards

1 - 35

96.

Which of the following is an argument against using historical cost in accounting? a. Fair values are more relevant. b. Historical costs are based on an exchange transaction. c. Historical costs are reliable. d. Fair values are subjective.

97.

When is revenue generally recognized? a. When cash is received. b. When the warranty expires. c. When production is completed. d. When the sale occurs.

98.

Which of the following are the two components of the revenue recognition principle? a. Cash is received and the amount is material. b. Recognition occurs when and earned realized or realizable. c. Production is complete and there is an active market for the product. d. Cash is realized or realizable and production is complete.

99.

Which of the following practices may not be an acceptable deviation from recognizing revenue at the point of sale? a. Upon receipt of cash. b. During production. c. Upon receipt of order. d. End of production.

100.

Which of the following is not a required component of financial statements prepared in accordance with generally accepted accounting principles? a. President's letter to shareholders. b. Balance sheet. c. Income statement. d. Notes to financial statements.

101.

What is the general approach as to when product costs are recognized as expenses? a. In the period when the expenses are paid. b. In the period when the expenses are incurred. c. In the period when the vendor invoice is received. d. In the period when the related revenue is recognized.

102.

Not adjusting the amounts reported in the financial statements for inflation is an example of which basic principle of accounting? a. Economic entity. b. Going concern. c. Historical cost. d. Full disclosure.

103.

Recognition of expense related to amortization of an intangible asset illustrates which principle of accounting?


Test Bank for Intermediate Accounting, Fourteenth Edition

1 - 36

a. b. c. d.

Expense recognition. Full disclosure. Revenue recognition. Historical cost.

104.

When should an expenditure be recorded as an asset rather than an expense? a. Never. b. Always. c. If the amount is material. d. When future benefit exits.

105.

Which accounting assumption or principle is being violated if a company reports its corporate headquarter building at its fair value on the balance sheet? a. Going concern. b. Monetary unit. c. Historical cost. d. Full disclosure.

106.

Which accounting assumption or principle is being violated if a company is a party to major litigation that it may lose and decides not to include the information in the financial statements because it may have a negative impact on the company's stock price? a. Full disclosure. b. Going concern. c. Historical cost. d. Expense recognition.

107.

Which assumption or principle requires that all information significant enough to affect a decision of reasonably informed users should be reported in the financial statements? a. Matching. b. Going concern. c. Historical cost. d. Full disclosure.

108.

A company has a factory building that originally cost the company $250,000. The current fair value of the factory building is $3 million. The president would like to report the difference as a gain. The write-up would represent a violation of which accounting assumption or principle? a. Revenue recognition. b. Going concern. c. Historical cost. d. Monetary unit.

109.

Which of the following is a constraint in presenting financial information? a. Industry practice. b. Full disclosure. c. Relevance. d. Consistency.


Financial Accounting and Accounting Standards

110.

All of the following represent costs of providing financial information except a. preparing. b. disseminating. c. accessing capital. d. auditing.

1 - 37


1 - 38

Test Bank for Intermediate Accounting, Fourteenth Edition

111.

Which of the following are benefits of providing financial information? a. Potential litigation. b. Auditing. c. Disclosure to competition. d. Improved allocation of resources.

112.

Where is materiality not used in providing financial information? a. Applying the revenue recognition principle. b. Determining what items to include in the financial statements. c. Applying the going concern assumption. d. Determining the level of disclosure.

113.

What is prudence or conservatism? a. Understating assets and net income. b. When in doubt, recognizing the option that is least likely to overstate assets and income. c. Recognizing the option that is least likely to overstate assets and income. d. Recognizing revenue when earned and realized.

114.

Expensing the cost of copy paper when the paper is acquired is an example of which constraint? a. Materiality. b. Cost. c. Conservatism. d. Industry practices.

115.

Which of the following statements concerning the cost-benefit relationship is not true? a. Business reporting should exclude information outside of management's expertise. b. Management should not be required to report information that would significantly harm the company's competitive position. c. Management should not be required to provide forecasted financial information. d. If needed by financial statement users, management should gather information not included in the financial statements that would not otherwise be gathered for internal use.

116.

Which of the following relates to both relevance and faithful representation? a. Cost constraint b. Predictive value c. Verifiability d. Neutrality

117.

Charging off the cost of a wastebasket with an estimated useful life of 10 years as an expense of the period when purchased is an example of the application of the a. consistency characteristic. b. expense recognition principle. c. materiality characteristic. d. historical cost principle.


Financial Accounting and Accounting Standards

1 - 39


1 - 40

Test Bank for Intermediate Accounting, Fourteenth Edition

118.

Which of the following statements about materiality is not correct? a. An item must make a difference or it need not be disclosed. b. Materiality is a matter of relative size or importance. c. An item is material if its inclusion or omission would influence or change the judgment of a reasonable person. d. All of these are correct statements about materiality.

119.

Which of the following are considered pervasive constraints by Statement of Financial Accounting Concepts No. 2? a. Cost-constraint relationship and conservatism b. Timeliness and feedback value c. Conservatism and verifiability d. Materiality and cost-constraint relationship

120.

The basic accounting concept that refers to the tendency of accountants to resolve uncertainty in favor of understating assets and revenues and overstating liabilities and expenses is known as a. prudence or conservatism. b. the materiality constraint. c. the substance over form principle. d. the industry practices constraint.

121.

Following the peculiar nature of some business concerns, which sometimes requires departure from basic theory is known as a. the economic entity assumption. b. industry practices. c. the cost constraint. d. the going concern assumption.

122.

Trade-offs between the characteristics that make information useful may be necessary or beneficial. Issuance of interim financial statements is an example of a trade-off between a. relevance and faithful representation. b. faithful representation and periodicity. c. timeliness and materiality. d. understandability and timeliness.

123.

Allowing firms to estimate rather than physically count inventory at interim (quarterly) periods is an example of a trade-off between a. verifiability and faithful representation. b. faithful representation and comparability. c. timeliness and verifiability. d. neutrality and consistency.

P

124. In matters of doubt and great uncertainty, accounting issues should be resolved by choosing the alternative that has the least favorable effect on net income, assets, and owners' equity. This guidance comes from a. the cost constraint.


Financial Accounting and Accounting Standards

b. the industry practices constraint. c. prudence or conservatism. d. the full disclosure principle.

1 - 41


1 - 42

Test Bank for Intermediate Accounting, Fourteenth Edition

Multiple Choice Answers—Conceptual Solutions to those Multiple Choice questions for which the answer is “none of these.” 55. a company changes its inventory method every few years in order to maximize reported income (other answers are possible). 58. comparability. 62. change in equity of an entity during a period from transactions and other events and circumstances from nonowner sources. 82. going concern assumption. 87. an exchange has taken place and the earnings process is virtually complete. MULTIPLE CHOICE—CPA Adapted 125.

According to the FASB's conceptual framework, predictive value is an ingredient of Relevance Faithful Representation a. Yes No b. Yes Yes c. No Yes d. No No

126.

According to the FASB's conceptual framework, which of the following relates to both relevance and faithful representation? Comparability Neutrality a. Yes Yes b. Yes No c. No Yes d. No No

127.

The FASB's conceptual framework classifies gains and losses based on whether they are related to an entity's major ongoing or central operations. These gains or losses may be classified as Nonoperating Operating a. Yes No b. Yes Yes c. No Yes d. No No

128.

According to the FASB's conceptual framework, earnings a. is the same as comprehensive income. b. excludes certain gains and losses that are included in comprehensive income. c. includes certain gains and losses that are excluded from comprehensive income. d. includes certain losses that are excluded from comprehensive income.

129.

According to the FASB's conceptual framework, comprehensive income includes which of the following? Operating Income Investments by Owners


Financial Accounting and Accounting Standards

a. b. c. d.

Yes Yes No No

1 - 43

No Yes Yes No

130.

According to the FASB's conceptual framework, the calculation of comprehensive income includes which of the following? Income from Distributions Continuing Operations to Owners a. No No b. Yes No c. Yes Yes d. No Yes

131.

According to the FASB's conceptual framework, comprehensive income includes which of the following? Gross Margin Operating Income a. No Yes b. No No c. Yes No d. Yes Yes

132.

Under Statements of Financial Accounting Concepts, comprehensive income includes which of the following? Gains Gross Margin a. No No b. No Yes c. Yes No d. YesYes


1 - 44

133.

Test Bank for Intermediate Accounting, Fourteenth Edition

According to the FASB's conceptual framework, the process of reporting an item in the financial statements of an entity is a. recognition. b. realization. c. allocation. d. matching.

Multiple Choice Answers—CPA Adapted

IFRS QUESTIONS True / False 1. The conceptual framework underlying U.S. GAAP is similar to that underlying IFRS. 2. The FASB conceptual framework specifically identifies accrual basis accounting as one of its fundamental assumptions. 3. One of two assumptions made by the IASB conceptual framework is that the reporting entity is a going concern. 4. One of the challenges in developing a common conceptual framework will be to agree on how the framework should be organized since the FASB and IASB conceptual frameworks are organized in very different ways. 5. One issue that the IASB and FASB must resolve in developing a common conceptual framework is how control should be defined with regard to the definition of an asset. Answers to True / False questions: Multiple Choice Questions: 1.

Which of the following statements regarding the IASB and FASB conceptual frameworks is not correct? a. The existing IASB and FASB conceptual frameworks are organized in similar ways. b. The two assumptions of the IASB framework are that the financial statements are prepared on an accrual basis and that the reporting entity is a going concern. c. The FASB and IASB agree that the sole objective of financial reporting is to provide users with information that is useful for decision-making. d. The FASB conceptual framework discusses the concept of accrual basis accounting in detail, but does not specifically identity it as an assumption.

2.

The issues which the FASB and IASB must address in developing a common conceptual framework include all of the following except:


Financial Accounting and Accounting Standards

1 - 45

a. Should the common framework lead to standards that are principles-based or rulesbased? b. Should the role of financial reporting focus on stewardship as well as providing information to assist users in decision making? c. Should the characteristic of reliability be traded-off in favor of information that is verifiable? d. Should a single measurement method such as historical cost be used? Answers to Multiple Choice:

Short Answer: 1. What two assumptions are central to the IFRS conceptual framework? 2. Do the IFRS and U.S. GAAP conceptual frameworks differ in terms of the role of financial reporting? Explain.

CHAPTER 3 THE ACCOUNTING INFORMATION SYSTEM IFRS questions are available at the end of this chapter. TRUE/FALSE 1.

A ledger is where the company initially records transactions and selected other events.

2.

Nominal (temporary) accounts are revenue, expense, and dividend accounts and are periodically closed.

3.

Real (permanent) accounts are revenue, expense, and dividend accounts and are periodically closed.

4.

An example of an internal event would be a flood that destroyed a portion of a company's inventory.

5.

All liability and stockholders’ equity accounts are increased on the credit side and decreased on the debit side.

6.

In general, debits refer to increases in account balances, and credits refer to decreases.

7.

The first step in the accounting cycle is the journalizing of transactions and selected other events.


1 - 46

Test Bank for Intermediate Accounting, Fourteenth Edition

8.

One purpose of a trial balance is to prove that debits and credits of an equal amount are in the general ledger.

9.

A general journal chronologically lists transactions and other events, expressed in terms of debits and credits to accounts.

10.

If a company fails to post one of its journal entries to its general ledger, the trial balance will not show an equal amount of debit and credit balance accounts.

11.

Adjusting entries for prepayments record the portion of the prepayment that represents the expense incurred or the revenue earned in the current accounting period.

12.

An adjustment for wages expense, earned but unpaid at year end, is an example of an accrued expense.

13.

The book value of any depreciable asset is the difference between its cost and its salvage value.

14.

The ending retained earnings balance is reported on both the retained earnings statement and the balance sheet.

15.

The post-closing trial balance consists of asset, liability, owners' equity, revenue and expense accounts.

16.

All revenues, expenses, and the dividends account are closed through the Income Summary account.

17.

It is not necessary to post the closing entries to the ledger accounts because new revenue and expense accounts will be opened in the subsequent accounting period.

*18.

The accrual basis recognizes revenue when earned and expenses in the period when cash is paid.

*19.

Reversing entries are made at the end of the accounting cycle to correct errors in the original recording of transactions.

*20.

An adjusted trial balance that shows equal debit and credit columnar totals proves the accuracy of the adjusting entries.

True / False Answers — Conceptual

MULTIPLE CHOICE—Conceptual


Financial Accounting and Accounting Standards

1 - 47

21.

Factors that shape an accounting information system include the a. nature of the business. b. size of the firm. c. volume of data to be handled. d. all of these.

22.

Maintaining a set of accounting records is a. optional. b. required by the Internal Revenue Service. c. required by the Foreign Corrupt Practices Act. d. required by the Internal Revenue Service and the Foreign Corrupt Practices Act.

23.

Debit always means a. right side of an account. b. increase. c. decrease. d. none of these.

24.

An accounting record into which the essential facts and figures in connection with all transactions are initially recorded is called the a. ledger. b. account. c. trial balance. d. none of these.


1 - 48

Test Bank for Intermediate Accounting, Fourteenth Edition

25.

A trial balance a. proves that debits and credits are equal in the ledger. b. supplies a listing of open accounts and their balances that are used in preparing financial statements. c. is normally prepared three times in the accounting cycle. d. all of these.

26.

Which of the following is a real (permanent) account? a. Goodwill b. Sales c. Accounts Receivable d. Both Goodwill and Accounts Receivable

27.

Which of the following is a nominal (temporary) account? a. Unearned Revenue b. Salary Expense c. Inventory d. Retained Earnings

28.

Nominal accounts are also called a. temporary accounts. b. permanent accounts. c. real accounts. d. none of these.

29.

The double-entry accounting system means a. Each transaction is recorded with two journal entries. b. Each item is recorded in a journal entry, then in a general ledger account. c. The dual effect of each transaction is recorded with a debit and a credit. d. More than one of the above.

30.

When a corporation pays a note payable and interest, a. the account notes payable will be increased. b. the account interest expense will be decreased. c. they will debit notes payable and interest expense. d. they will debit cash.

31.

Stockholders’ equity is not affected by all a. cash receipts. b. dividends. c. revenues. d. expenses.

32.

The debit and credit analysis of a transaction normally takes place a. before an entry is recorded in a journal. b. when the entry is posted to the ledger. c. when the trial balance is prepared.


Financial Accounting and Accounting Standards

d. at some other point in the accounting cycle.

1 - 49


1 - 50

Test Bank for Intermediate Accounting, Fourteenth Edition

33.

The accounting equation must remain in balance a. throughout each step in the accounting cycle. b. only when journal entries are recorded. c. only at the time the trial balance is prepared. d. only when formal financial statements are prepared.

34.

The difference between the accounting process and the accounting cycle is a. the accounting process results in the preparation of financial statements, whereas the accounting cycle is concerned with recording business transactions. b. the accounting cycle represents the steps taken to accomplish the accounting process. c. the accounting process represents the steps taken to accomplish the accounting cycle. d. merely semantic, because both concepts refer to the same thing.

35.

An optional step in the accounting cycle is the preparation of a. adjusting entries. b. closing entries. c. a statement of cash flows. d. a post-closing trial balance.

36.

Which of the following criteria must be met before an event or item should be recorded for accounting purposes? a. The event or item can be measured objectively in financial terms. b. The event or item is relevant and reliable. c. The event or item is an element. d. All of these must be met.

37.

Which of the following is a recordable event or item? a. Changes in managerial policy b. The value of human resources c. Changes in personnel d. None of these

38.

Which of the following is not an internal event? a. Depreciation b. Using raw materials in the production process c. Dividend declaration and subsequent payment d. All of these are internal transactions.

39.

External events do not include a. interaction between an entity and its environment. b. a change in the price of a good or service that an entity buys or sells, a flood or earthquake. c. improvement in technology by a competitor. d. using buildings and machinery in operations.

40.

A trial balance may prove that debits and credits are equal, but a. an amount could be entered in the wrong account.


Financial Accounting and Accounting Standards

b. a transaction could have been entered twice. c. a transaction could have been omitted. d. all of these.

1 - 51


1 - 52

Test Bank for Intermediate Accounting, Fourteenth Edition

41.

A general journal a. chronologically lists transactions and other events, expressed in terms of debits and credits. b. contains one record for each of the asset, liability, stockholders’ equity, revenue, and expense accounts. c. lists all the increases and decreases in each account in one place. d. contains only adjusting entries.

42.

A journal entry to record the sale of inventory on account will include a a. debit to inventory. b. debit to accounts receivable. c. debit to sales. d. credit to cost of goods sold.

43.

A journal entry to record a payment on account will include a a. debit to accounts receivable. b. credit to accounts receivable. c. debit to accounts payable. d. credit to accounts payable.

44.

A journal entry to record a receipt of rent revenue in advance will include a a. debit to rent revenue. b. credit to rent revenue. c. credit to cash. d. credit to unearned rent.

45.

Which of the following errors will cause an imbalance in the trial balance? a. Omission of a transaction in the journal. b. Posting an entire journal entry twice to the ledger. c. Posting a credit of $720 to Accounts Payable as a credit of $720 to Accounts Receivable. d. Listing the balance of an account with a debit balance in the credit column of the trial balance.

S

Which of the following is not a principal purpose of an unadjusted trial balance? a. It proves that debits and credits of equal amounts are in the ledger. b. It is the basis for any adjustments to the account balances. c. It supplies a listing of open accounts and their balances. d. It proves that debits and credits were properly entered in the ledger accounts.

S

47.

An adjusting entry should never include a. a debit to an expense account and a credit to a liability account. b. a debit to an expense account and a credit to a revenue account. c. a debit to a liability account and a credit to revenue account. d. a debit to a revenue account and a credit to a liability account.

48.

Which of the following is an example of an accrued expense?

46.


Financial Accounting and Accounting Standards

P

49.

1 - 53

a. Office supplies purchased at the beginning of the year and debited to an expense account. b. Property taxes incurred during the year, to be paid in the first quarter of the subsequent year. c. Depreciation expense d. Rent earned during the period, to be received at the end of the year Which of the following statements is associated with the accrual basis of accounting? a. The timing of cash receipts and disbursements is emphasized. b. A minimum amount of record keeping is required. c. This method is used less frequently by businesses than the cash method of accounting. d. Revenues are recognized in the period they are earned, regardless of the time period the cash is received.

P

An adjusting entry to record an accrued expense involves a debit to a(an): a. expense account and a credit to a prepaid account. b. expense account and a credit to Cash. c. expense account and a credit to a liability account. d. liability account and a credit to an expense account.

P

The failure to properly record an adjusting entry to accrue an expense will result in an: a. understatement of expenses and an understatement of liabilities. b. understatement of expenses and an overstatement of liabilities. c. understatement of expenses and an overstatement of assets. d. overstatement of expenses and an understatement of assets.

P

Which of the following properly describes a deferral? a. Cash is received after revenue is earned. b. Cash is received before revenue is earned. c. Cash is paid after expense is incurred. d. Cash is paid in the same time period that an expense is incurred.

P

The failure to properly record an adjusting entry to accrue a revenue item will result in an: a. understatement of revenues and an understatement of liabilities. b. overstatement of revenues and an overstatement of liabilities. c. overstatement of revenues and an overstatement of assets. d. understatement of revenues and an understatement of assets.

P

54.

The omission of the adjusting entry to record depreciation expense will result in an: a. overstatement of assets and an overstatement of owners' equity. b. understatement of assets and an understatement of owner's equity. c. overstatement of assets and an overstatement of liabilities. d. overstatement of liabilities and an understatement of owners' equity.

55.

Adjustments are often prepared a. after the balance sheet date, but dated as of the balance sheet date. b. after the balance sheet date, and dated after the balance sheet date. c. before the balance sheet date, but dated as of the balance sheet date.

50.

51.

52.

53.


1 - 54

Test Bank for Intermediate Accounting, Fourteenth Edition

d. before the balance sheet date, and dated after the balance sheet date. 56.

At the time a company prepays a cost a. it debits an asset account to show the service or benefit it will receive in the future. b. it debits an expense account to match the expense against revenues earned. c. its credits a liability account to show the obligation to pay for the service in the future. d. more than one of the above.


Financial Accounting and Accounting Standards

57.

How do these prepaid expenses expire? Rent a. With the passage of time b. With the passage of time c. Through use and consumption d. Through use and consumption

1 - 55

Supplies Through use and consumption With the passage of time Through use and consumption With the passage of time

58.

Recording the adjusting entry for depreciation has the same effect as recording the adjusting entry for a. an unearned revenue. b. a prepaid expense. c. an accrued revenue. d. an accrued expense.

59.

Unearned revenue on the books of one company is likely to be a. a prepaid expense on the books of the company that made the advance payment. b. an unearned revenue on the books of the company that made the advance payment. c. an accrued expense on the books of the company that made the advance payment. d. an accrued revenue on the books of the company that made the advance payment.

60.

To compute interest expense for an adjusting entry, the formula is (principal X annual rate X a fraction). The numerator and denominator of the fraction are: Numerator Denominator a. Length of time note has been outstanding 12 months b. Length of note 12 months c. Length of time until note matures Length of note d. Length of time note has been outstanding Length of note

61.

Adjusting entries are necessary to 1. obtain a proper matching of revenue and expense. 2. achieve an accurate statement of assets and equities. 3. adjust assets and liabilities to their fair market value. a. 1 b. 2 c. 3 d. 1 and 2

62.

Why are certain costs of doing business capitalized when incurred and then depreciated or amortized over subsequent accounting cycles? a. To reduce the federal income tax liability b. To aid management in cash-flow analysis c. To match the costs of production with revenues as earned d. To adhere to the accounting constraint of conservatism

63.

When an item of expense is paid and recorded in advance, it is normally called a(n) a. prepaid expense. b. accrued expense.


1 - 56

Test Bank for Intermediate Accounting, Fourteenth Edition

c. estimated expense. d. cash expense.


Financial Accounting and Accounting Standards

1 - 57

64.

When an item of revenue or expense has been earned or incurred but not yet collected or paid, it is normally called a(n) ____________ revenue or expense. a. prepaid b. adjusted c. estimated d. none of these

65.

When an item of revenue is collected and recorded in advance, it is normally called a(n) ___________ revenue. a. accrued b. prepaid c. unearned d. cash

66.

An accrued expense can best be described as an amount a. paid and currently matched with earnings. b. paid and not currently matched with earnings. c. not paid and not currently matched with earnings. d. not paid and currently matched with earnings.

67.

If, during an accounting period, an expense item has been incurred and consumed but not yet paid for or recorded, then the end-of-period adjusting entry would involve a. a liability account and an asset account. b. an asset or contra asset account and an expense account. c. a liability account and an expense account. d. a receivable account and a revenue account.

68.

Which of the following must be considered in estimating depreciation on an asset for an accounting period? a. The original cost of the asset b. Its useful life c. The decline of its fair market value d. Both the original cost of the asset and its useful life.

69.

Which of the following would not be a correct form for an adjusting entry? a. A debit to a revenue and a credit to a liability b. A debit to an expense and a credit to a liability c. A debit to a liability and a credit to a revenue d. A debit to an asset and a credit to a liability

70.

Year-end net assets would be overstated and current expenses would be understated as a result of failure to record which of the following adjusting entries? a. Expiration of prepaid insurance b. Depreciation of fixed assets c. Accrued wages payable d. All of these


1 - 58

71.

72.

Test Bank for Intermediate Accounting, Fourteenth Edition

A prepaid expense can best be described as an amount a. paid and currently matched with revenues. b. paid and not currently matched with revenues. c. not paid and currently matched with revenues. d. not paid and not currently matched with revenues. An accrued revenue can best be described as an amount a. collected and currently matched with expenses. b. collected and not currently matched with expenses. c. not collected and currently matched with expenses. d. not collected and not currently matched with expenses.

73.

An unearned revenue can best be described as an amount a. collected and currently matched with expenses. b. collected and not currently matched with expenses. c. not collected and currently matched with expenses. d. not collected and not currently matched with expenses.

74.

An adjusted trial balance a. is prepared after the financial statements are completed. b. proves the equality of the total debit balances and total credit balances of ledger accounts after all adjustments have been made. c. is a required financial statement under generally accepted accounting principles. d. cannot be used to prepare financial statements.

75.

Which type of account is always debited during the closing process? a. Dividends. b. Expense. c. Revenue. d. Retained earnings.

S

Which of the following statements best describes the purpose of closing entries? a. To faciliate posting and taking a trial balance. b. To determine the amount of net income or net loss for the period. c. To reduce the balances of revenue and expense accounts to zero so that they may be used to accumulate the revenues and expenses of the next period. d. To complete the record of various transactions that were started in a prior period.

P

If ending accounts receivable exceeds the beginning accounts receivable: a. cash collections during the period exceed the amount of revenue earned. b. net income for the period is less than the amount of cash basis income. c. no cash was collected during the period. d. cash collections during the year are less than the amount of revenue earned.

*78.

Under the cash basis of accounting, revenues are recorded a. when they are earned and realized. b. when they are earned and realizable. c. when they are earned.

76.

77.


Financial Accounting and Accounting Standards

1 - 59

d. when they are realized. *79.

When converting from cash basis to accrual-basis accounting, which of the following adjustments should be made to cash receipts from customers to determine accrual basis service revenue? a. Subtract ending accounts receivable. b. Subtract beginning unearned service revenue. c. Add ending accounts receivable. d. Add cash sales.

*80.

When converting from cash basis to accrual basis accounting, which of the following adjustments should be made to cash paid for operating expenses to determine accrual basis operating expenses? a. Add beginning accrued liabilities. b. Add beginning prepaid expense. c. Subtract ending prepaid expense. d. Subtract interest expense.

*81.

Reversing entries are 1. normally prepared for prepaid, accrued, and estimated items. 2. necessary to achieve a proper matching of revenue and expense. 3. desirable to exercise consistency and establish standardized procedures. a. 1 b. 2 c. 3 d. 1 and 2

*82.

Adjusting entries that should be reversed include those for prepaid or unearned items that a. create an asset or a liability account. b. were originally entered in a revenue or expense account. c. were originally entered in an asset or liability account. d. create an asset or a liability account and were originally entered in a revenue or expense account.

*83.

Adjusting entries that should be reversed include a. all accrued revenues. b. all accrued expenses. c. those that debit an asset or credit a liability. d. all of these.

S

*84. A reversing entry should never be made for an adjusting entry that a. accrues unrecorded revenue. b. adjusts expired costs from an asset account to an expense account. c. accrues unrecorded expenses. d. adjusts unexpired costs from an expense account to an asset account.


1 - 60 S

Test Bank for Intermediate Accounting, Fourteenth Edition

*85. The worksheet for Sharko Co. consisted of five pairs of debit and credit columns. The dollar amount of one item appeared in both the credit column of the income statement section and the debit column of the balance sheet section. That item is a. net income for the period. b. beginning inventory. c. cost of goods sold. d. Net loss for the period.


Financial Accounting and Accounting Standards

1 - 61

MULTIPLE CHOICE—Computational 86.

Maso Company recorded journal entries for the issuance of common stock for $80,000, the payment of $26,000 on accounts payable, and the payment of salaries expense of $42,000. What net effect do these entries have on owners’ equity? a. Increase of $80,000. b. Increase of $54,000. c. Increase of $38,000. d. Increase of $12,000.

87.

Mune Company recorded journal entries for the declaration of $100,000 of dividends, the $64,000 increase in accounts receivable for services rendered, and the purchase of equipment for $42,000. What net effect do these entries have on owners’ equity? a. Decrease of $142,000. b. Decrease of $78,000. c. Decrease of $36,000. d. Increase of $22,000.

88.

Pappy Corporation received cash of $18,000 on September 1, 2012 for one year’s rent in advance and recorded the transaction with a credit to Unearned Rent Revenue. The December 31, 2012 adjusting entry is a. debit Rent Revenue and credit Unearned Rent Revenue, $6,000. b. debit Rent Revenue and credit Unearned Rent Revenue, $12,000. c. debit Unearned Rent Revenue and credit Rent Revenue, $6,000. d. debit Cash and credit Unearned Rent Revenue, $12,000.

89.

Panda Corporation paid cash of $30,000 on June 1, 2012 for one year’s rent in advance and recorded the transaction with a debit to Prepaid Rent. The December 31, 2012 adjusting entry is a. debit Prepaid Rent and credit Rent Expense, $12,500. b. debit Prepaid Rent and credit Rent Expense, $17,500. c. debit Rent Expense and credit Prepaid Rent, $17,500. d. debit Prepaid Rent and credit Cash, $12,500.

90.

Tate Company purchased equipment on November 1, 2012 and gave a 3-month, 9% note with a face value of $40,000. The December 31, 2012 adjusting entry is a. debit Interest Expense and credit Interest Payable, $3,600. b. debit Interest Expense and credit Interest Payable, $900. c. debit Interest Expense and credit Cash, $600. d. debit Interest Expense and credit Interest Payable, $600.

91.

Brown Company's account balances at December 31, 2012 for Accounts Receivable and the related Allowance for Doubtful Accounts are $920,000 debit and $1,400 credit, respectively. From an aging of accounts receivable, it is estimated that $25,000 of the


1 - 62

Test Bank for Intermediate Accounting, Fourteenth Edition

December 31 receivables will be uncollectible. The necessary adjusting entry would include a credit to the allowance account for a. $25,000. b. $26,400. c. $23,600. d. $1,400. 92.

Chen Company's account balances at December 31, 2012 for Accounts Receivable and the Allowance for Doubtful Accounts are $480,000 debit and $900 credit. Sales during 2012 were $1,350,000. It is estimated that 1% of sales will be uncollectible. The adjusting entry would include a credit to the allowance account for a. $14,400. b. $13,500. c. $12,600. d. $4,800.

93.

Starr Corporation loaned $150,000 to another corporation on December 1, 2012 and received a 3-month, 8% interest-bearing note with a face value of $150,000. What adjusting entry should Starr make on December 31, 2012? a. Debit Interest Receivable and credit Interest Revenue, $3,000. b. Debit Cash and credit Interest Revenue, $1,000. c. Debit Interest Receivable and credit Interest Revenue, $1,000. d. Debit Cash and credit Interest Receivable, $3,000.

94.

A company receives interest on a $40,000, 8%, 5-year note receivable each April 1. At December 31, 2012, the following adjusting entry was made to accrue interest receivable: Interest Receivable ................................................................ Interest Revenue ........................................................

2,400 2,400


Financial Accounting and Accounting Standards

1 - 63

Assuming that the company does not use reversing entries, what entry should be made on April 1, 2013 when the annual interest payment is received? a. Cash ....................................................................................... 800 Interest Revenue ........................................................ 800 b. Cash ....................................................................................... 2,400 Interest Receivable .................................................... 2,400 c. Cash ....................................................................................... 3,200 Interest Receivable .................................................... 2,400 Interest Revenue ........................................................ 800 d. Cash ....................................................................................... 3,200 Interest Revenue ........................................................ 3,200 *95.

A company receives interest on a $40,000, 8%, 5-year note receivable each April 1. At December 31, 2012, the following adjusting entry was made to accrue interest receivable: Interest Receivable ................................................................ Interest Revenue ........................................................

2,400 2,400

Assuming that the company does use reversing entries, what entry should be made on April 1, 2013 when the annual interest payment is received? a. Cash ....................................................................................... 800 Interest Revenue ........................................................ 800 b. Cash ....................................................................................... 2,400 Interest Receivable .................................................... 2,400 c. Cash ..................................................................................... 3,200 Interest Receivable .................................................... 2,400 Interest Revenue ........................................................ 800 d. Cash ....................................................................................... 3,200 Interest Revenue......................................................... 3,200 96.

Murphy Company sublet a portion of its warehouse for five years at an annual rental of $30,000, beginning on May 1, 2012. The tenant, Sheri Charter, paid one year's rent in advance, which Murphy recorded as a credit to Unearned Rent Revenue. Murphy reports on a calendar-year basis. The adjustment on December 31, 2012 for Murphy should be a. No entry b. Unearned Rent Revenue ....................................................... 10,000 Rent Revenue ............................................................ 10,000 c. Rent Revenue ........................................................................ 10,000 Unearned Rent Revenue ........................................... 10,000 d. Unearned Rent Revenue ....................................................... 20,000 Revenue Revenue....................................................... 20,000

97.

During the first year of Wilkinson Co.'s operations, all purchases were recorded as assets. Supplies in the amount of $25,800 were purchased. Actual year-end supplies amounted to $8,600. The adjusting entry for store supplies will a. increase net income by $17,200. b. increase expenses by $17,200. c. decrease supplies by $8,600.


1 - 64

Test Bank for Intermediate Accounting, Fourteenth Edition

d. debit Accounts Payable for $8,600.


Financial Accounting and Accounting Standards

98.

*99.

1 - 65

Big-Mouth Frog Corporation had revenues of $300,000, expenses of $180,000, and dividends of $45,000. When Income Summary is closed to Retained Earnings, the amount of the debit or credit to Retained Earnings is a a. debit of $75,000. b. debit of $120,000. c. credit of $75,000. d. credit of $120,000. The income statement of Dolan Corporation for 2012 included the following items: Interest revenue $131,000 Salaries and wages expense 170,000 Insurance expense 15,200

The following balances have been excerpted from Dolan Corporation's balance sheets: December 31, 2012 December 31, 2011 Interest receivable $18,200 $15,000 Salaries and wages payable 17,800 8,400 Prepaid insurance 2,200 3,000 The cash received for interest during 2012 was a. $112,800. b. $127,800. c. $131,000. d. $134,200. *100. The income statement of Dolan Corporation for 2012 included the following items: Interest revenue $131,000 Salaries and wages expense 170,000 Insurance expense 15,200 The following balances have been excerpted from Dolan Corporation's balance sheets: December 31, 2012 December 31, 2011 Interest receivable $18,200 $15,000 Salaries and wages payable 17,800 8,400 Prepaid insurance 2,200 3,000 The cash paid for salaries during 2012 was a. $179,400. b. $160,600. c. $161,600. d. $187,800. *101. The income statement of Dolan Corporation for 2012 included the following items: Interest revenue $131,000 Salaries and wages expense 170,000 Insurance expense 15,200


1 - 66

Test Bank for Intermediate Accounting, Fourteenth Edition


Financial Accounting and Accounting Standards

1 - 67

The following balances have been excerpted from Dolan Corporation's balance sheets: December 31, 2012 December 31, 2011 Interest receivable $18,200 $15,000 Salaries and wages payable 17,800 8,400 Prepaid insurance 2,200 3,000 The cash paid for insurance premiums during 2012 was a. $13,000. b. $12,200. c. $16,000. d. $14,400. *102. Olsen Company paid or collected during 2012 the following items: Insurance premiums paid $ 20,800 Interest collected 67,800 Salaries paid 240,400 The following balances have been excerpted from Olsen's balance sheets: December 31, 2012 Prepaid insurance $ 2,400 Interest receivable 7,400 Salaries and wages payable 24,600

December 31, 2011 $ 3,000 5,800 21,200

The insurance expense on the income statement for 2012 was a. $15,400. b. $20,200. c. $21,400. d. $26,200. *103. Olsen Company paid or collected during 2012 the following items: Insurance premiums paid $ 20,800 Interest collected 67,800 Salaries paid 240,400 The following balances have been excerpted from Olsen's balance sheets: December 31, 2012 Prepaid insurance $ 2,400 Interest receivable 7,400 Salaries and wages payable 24,600 The interest revenue on the income statement for 2012 was a. $54,600. b. $66,200. c. $69,400. d. $81,000.

December 31, 2011 $ 3,000 5,800 21,200


1 - 68

Test Bank for Intermediate Accounting, Fourteenth Edition


Financial Accounting and Accounting Standards

1 - 69

*104. Olsen Company paid or collected during 2012 the following items: Insurance premiums paid $ 20,800 Interest collected 67,800 Salaries paid 240,400 The following balances have been excerpted from Olsen's balance sheets: December 31, 2012 Prepaid insurance $ 2,400 Interest receivable 7,400 Salaries and wages payable 24,600

December 31, 2011 $ 3,000 5,800 21,200

Salaries expense on the income statement for 2012 was a. $194,600. b. $237,000. c. $243,800. d. $286,200. *105. The Supplies account had a balance at the beginning of year 3 of $8,000 (before the reversing entry). Payments for purchases of supplies during year 3 amounted to $50,000 and were recorded as expense. A physical count at the end of year 3 revealed supplies costing $9,500 were on hand. Reversing entries are used by this company. The required adjusting entry at the end of year 3 will include a debit to: a. Supplies Expense for $1,500. b. Supplies for $1,500. c. Supplies Expense for $48,500. d. Supplies for $9,500. *106. At the end of 2012, Drew Company made four adjusting entries for the following items: 1. Depreciation expense, $25,000. 2. Expired insurance, $2,200 (originally recorded as prepaid insurance.) 3. Interest payable, $6,000. 4. Rent receivable, $10,000. In the normal situation, to facilitate subsequent entries, the adjusting entry or entries that may be reversed is (are) a. Entry No. 3. b. Entry No. 4. c. Entry No. 3 and No. 4. d. Entry No. 2, No. 3 and No. 4. *107. Garcia Corporation received cash of $24,000 on August 1, 2012 for one year's rent in advance and recorded the transaction with a credit to Rent Revenue. The December 31, 2012 adjusting entry is a. debit Rent Revenue and credit Unearned Rent Revenue, $10,000. b. debit Rent Revenue and credit Unearned Rent Revenue, $14,000. c. debit Unearned Rent Revenue and credit Rent Revenue, $10,000. d. debit Cash and credit Unearned Rent Revenue, $14,000.


1 - 70

Test Bank for Intermediate Accounting, Fourteenth Edition

*108. Lopez Company received $9,600 on April 1, 2012 for one year's rent in advance and recorded the transaction with a credit to a nominal account. The December 31, 2012 adjusting entry is a. debit Rent Revenue and credit Unearned Rent Revenue, $2,400. b. debit Rent Revenue and credit Unearned Rent Revenue, $7,200. c. debit Unearned Rent Revenue and credit Rent Revenue, $2,400. d. debit Unearned Rent Revenue and credit Rent Revenue, $7,200. *109. Gibson Company paid $6,000 on June 1, 2012 for a two-year insurance policy and recorded the entire amount as Insurance Expense. The December 31, 2012 adjusting entry is a. debit Insurance Expense and credit Prepaid Insurance, $1,750. b. debit Insurance Expense and credit Prepaid Insurance, $4,250. c. debit Prepaid Insurance and credit Insurance Expense, $1,750 d. debit Prepaid Insurance and credit Insurance Expense, $4,250. Multiple Choice Answers—Computational

MULTIPLE CHOICE—CPA Adapted 110.

On September 1, 2012, Lowe Co. issued a note payable to National Bank in the amount of $900,000, bearing interest at 12%, and payable in three equal annual principal payments of $300,000. On this date, the bank's prime rate was 11%. The first payment for interest and principal was made on September 1, 2013. At December 31, 2013, Lowe should record accrued interest payable of a. $36,000. b. $33,000. c. $24,000. d. $22,000.

111.

Eaton Co. sells major household appliance service contracts for cash. The service contracts are for a one-year, two-year, or three-year period. Cash receipts from contracts are credited to Unearned Service Revenue. This account had a balance of $3,600,000 at December 31, 2012 before year-end adjustment. Service contract costs are charged as incurred to the Service Contract Expense account, which had a balance of $900,000 at December 31, 2012. Service contracts still outstanding at December 31, 2012 expire as follows: During 2013 $760,000 During 2014 1,140,000 During 2015 700,000 What amount should be reported as Unearned Service Revenue in Eaton's December 31, 2012 balance sheet? a. $2,700,000.


Financial Accounting and Accounting Standards

1 - 71

b. $2,600,000. c. $1,700,000. d. $1,000,000. 112.

In November and December 2012, Lane Co., a newly organized magazine publisher, received $75,000 for 1,000 three-year subscriptions at $25 per year, starting with the January 2013 issue. Lane included the entire $75,000 in its 2012 income tax return. What amount should Lane report in its 2012 income statement for subscriptions revenue? a. $0. b. $4,167. c. $25,000. d. $75,000.

113.

On June 1, 2012, Nott Corp. loaned Horn $600,000 on a 12% note, payable in five annual installments of $120,000 beginning January 2, 2013. In connection with this loan, Horn was required to deposit $5,000 in a noninterest-bearing escrow account. The amount held in escrow is to be returned to Horn after all principal and interest payments have been made. Interest on the note is payable on the first day of each month beginning July 1, 2012. Horn made timely payments through November 1, 2012. On January 2, 2013, Nott received payment of the first principal installment plus all interest due. At December 31, 2012, Nott's interest receivable on the loan to Horn should be a. $0. b. $6,000. c. $12,000. d. $18,000.

114.

Allen Corp.'s liability account balances at June 30, 2013 included a 10% note payable in the amount of $3,000,000. The note is dated October 1, 2011 and is payable in three equal annual payments of $1,000,000 plus interest. The first interest and principal payment was made on October 1, 2012. In Allen's June 30, 2013 balance sheet, what amount should be reported as accrued interest payable for this note? a. $225,000. b. $150,000. c. $75,000. d. $50,000.

115.

Colaw Co. pays all salaried employees on a biweekly basis. Overtime pay, however, is paid in the next biweekly period. Colaw accrues salaries expense only at its December 31 year end. Data relating to salaries earned in December 2012 are as follows: Last payroll was paid on 12/26/12, for the 2-week period ended 12/26/12. Overtime pay earned in the 2-week period ended 12/26/12 was $15,000. Remaining work days in 2012 were December 29, 30, 31, on which days there was no overtime. The recurring biweekly salaries total $270,000.


Test Bank for Intermediate Accounting, Fourteenth Edition

1 - 72

Assuming a five-day work week, Colaw should record a liability at December 31, 2012 for accrued salaries of a. b. c. d. 116.

Tolan Corp.'s trademark was licensed to Eddy Co. for royalties of 15% of sales of the trademarked items. Royalties are payable semiannually on March 15 for sales in July through December of the prior year, and on September 15 for sales in January through June of the same year. Tolan received the following royalties from Eddy: March 15 September 15 2011 $5,000 $7,500 2012 6,000 8,500 Eddy estimated that sales of the trademarked items would total $30,000 for July through December 2012. In Tolan's 2012 income statement, the royalty revenue should be a. b. c. d.

117.

$81,000. $96,000. $162,000. $177,000.

$13,000. $14,500. $19,000. $20,500.

At December 31, 2012, Sue’s Boutique had 1,000 gift certificates outstanding, which had been sold to customers during 2012 for $75 each. Sue’s operates on a gross profit of 60% of its sales. What amount of revenue pertaining to the 1,000 outstanding gift certificates should be deferred at December 31, 2012? a. $0. b. $30,000. c. $45,000. d. $75,000.

*118. Compared to the accrual basis of accounting, the cash basis of accounting overstates income by the net increase during the accounting period of the Accounts Receivable Accrued Expenses Payable a. No No b. No Yes c. Yes No d. Yes Yes *119. Gregg Corp. reported revenue of $1,250,000 in its accrual basis income statement for the year ended June 30, 2013. Additional information was as follows: Accounts receivable June 30, 2012 $400,000 Accounts receivable June 30, 2013 530,000 Uncollectible accounts written off during the fiscal year 15,000 Under the cash basis, Gregg should report revenue of a. $835,000.


Financial Accounting and Accounting Standards

1 - 73

b. $850,000. c. $1,105,000. d. $1,135,000. *120. Jim Yount, M.D., keeps his accounting records on the cash basis. During 2013, Dr. Yount collected $300,000 from his patients. At December 31, 2012, Dr. Yount had accounts receivable of $40,000. At December 31, 2013, Dr. Yount had accounts receivable of $70,000 and unearned revenue of $10,000. On the accrual basis, how much was Dr. Yount's patient service revenue for 2013? a. $260,000. b. $320,000. c. $330,000. d. $340,000. *121. The following information is available for Ace Company for 2012: Disbursements for purchases Increase in trade accounts payable Decrease in merchandise inventory Cost of goods sold for 2012 was a. $1,300,000. b. $1,220,000. c. $1,100,000. d. $1,020,000. Multiple Choice Answers—CPA Adapted

$1,160,000 100,000 40,000


1 - 74

Test Bank for Intermediate Accounting, Fourteenth Edition

IFRS QUESTIONS Short Answer: 1. Are all international companies subject to the same internal control standards? Explain.

2. What are some of the consequences of international differences in internal control standards?

CHAPTER 4 INCOME STATEMENT AND RELATED INFORMATION IFRS questions are available at the end of this chapter. TRUE-FALSE—Conceptual 1. The income statement is useful for helping to assess the risk or uncertainty of achieving future cash flows. 2. A strength of the income statement as compared to the balance sheet is that items that cannot be measured reliably can be reported in the income statement. 3. Earnings management generally makes income statement information more useful for predicting future earnings and cash flows. 4. The transaction approach of income measurement focuses on the income-related activities that have occurred during the period. 5. Companies frequently report income tax expense as the last item before net income on a single-step income statement. 6. Both revenues and gains increase both net income and owners’ equity. 7. Use of a multiple-step income statement will result in the company reporting a higher net income than if they used a single-step income statement. 8. The primary advantage of the multiple-step format lies in the simplicity of presentation and the absence of any implication that one type of revenue or expense item has priority over another. 9. Gross profit and income from operations are reported on a multiple-step but not a single-step income statement.


Financial Accounting and Accounting Standards

1 - 75

10. The accounting profession has adopted a current operating performance approach to income reporting. 11. Companies report the results of operations of a component of a business that will be disposed of separately from continuing operations. 12. Gains or losses from exchange or translation of foreign currencies are reported as extraordinary items. 13. Discontinued operations, extraordinary items, and unusual gains and losses are all reported net of tax in the income statement. 14. Intraperiod tax allocation relates the income tax expense of the period to the specific items that give rise to the amount of the tax provision. 15. A company that reports a discontinued operation or an extraordinary item has the option of reporting per share amounts for these items. 16. Dividends declared on common and preferred stock are subtracted from net income in the computation of earnings per share. 17. Prior period adjustments can either be added or subtracted in the Retained Earnings Statement. 18. Companies only restrict retained earnings to comply with contractual requirements or current necessity. 19. Comprehensive income includes all changes in equity during a period except those resulting from distributions to owners. 20. The components of other comprehensive income can be reported in a statement of stockholders’ equity. True False Answers—Conceptual

MULTIPLE CHOICE—Conceptual 21.

The major elements of the income statement are a. revenue, cost of goods sold, selling expenses, and general expense. b. operating section, nonoperating section, discontinued operations, extraordinary items, and cumulative effect. c. revenues, expenses, gains, and losses. d. all of these.


1 - 76

Test Bank for Intermediate Accounting, Fourteenth Edition

22.

Information in the income statement helps users to a. evaluate the past performance of the enterprise. b. provide a basis for predicting future performance. c. help assess the risk or uncertainty of achieving future cash flows. d. all of these.

23.

Limitations of the income statement include all of the following except a. items that cannot be measured reliably are not reported. b. only actual amounts are reported in determining net income. c. income measurement involves judgment. d. income numbers are affected by the accounting methods employed.

S

24.

S

25.

Which of the following would represent the least likely use of an income statement prepared for a business enterprise? a. Use by customers to determine a company's ability to provide needed goods and services. b. Use by labor unions to examine earnings closely as a basis for salary discussions. c. Use by government agencies to formulate tax and economic policy. d. Use by investors interested in the financial position of the entity. The income statement reveals a. resources and equities of a firm at a point in time. b. resources and equities of a firm for a period of time. c. net earnings (net income) of a firm at a point in time. d. net earnings (net income) of a firm for a period of time.

26.

The income statement information would help in which of the following tasks? a. Evaluate the liquidity of a company. b. Evaluate the solvency of a company c. Estimate future cash flows d. Estimate future financial flexibility

27.

Which of the following is an example of managing earnings down? a. Changing estimated bad debts from 3 percent to 2.5 percent of sales. b. Revising the estimated life of equipment from 10 years to 8 years. c. Not writing off obsolete inventory. d. Reducing research and development expenditures.

28.

Which of the following is an example of managing earnings up? a. Decreasing estimated salvage value of equipment. b. Writing off obsolete inventory. c. Underestimating warranty claims. d. Accruing a contingent liability for an ongoing lawsuit.

29.

What might a manager do during the last quarter of a fiscal year if she wanted to improve current annual net income? a. Increase research and development activities. b. Relax credit policies for customers.


Financial Accounting and Accounting Standards

1 - 77

c. Delay shipments to customers until after the end of the fiscal year. d. Delay purchases from suppliers until after the end of the fiscal year. 30.

What might a manager do during the last quarter of a fiscal year if she wanted to decrease current annual net income? a. Delay shipments to customers until after the end of the fiscal year. b. Relax credit policies for customers. c. Pay suppliers all amounts owed. d. Delay purchases from suppliers until after the end of the fiscal year.

31.

Which of the following is an advantage of the single-step income statement over the multiple-step income statement? a. It reports gross profit for the year. b. Expenses are classified by function. c. It matches costs and expenses with related revenues. d. It does not imply that one type of revenue or expense has priority over another.

32.

The single-step income statement emphasizes a. the gross profit figure. b. total revenues and total expenses. c. extraordinary items and accounting changes more than these are emphasized in the multiple-step income statement. d. the various components of income from continuing operations.

33.

Which of the following is an acceptable method of presenting the income statement? a. A single-step income statement b. A multiple-step income statement c. A consolidated statement of income d. All of these

34.

Which of the following is not a generally practiced method of presenting the income statement? a. Including prior period adjustments in determining net income b. The single-step income statement c. The consolidated statement of income d. Including gains and losses from discontinued operations of a component of a business in determining net income

35.

The occurrence which most likely would have no effect on 2012 net income (assuming that all amounts involved are material) is the a. sale in 2012 of an office building contributed by a stockholder in 1983. b. collection in 2012 of a receivable from a customer whose account was written off in 2011 by a charge to the allowance account. c. settlement based on litigation in 2012 of previously unrecognized damages from a serious accident which occurred in 2010. d. worthlessness determined in 2012 of stock purchased on a speculative basis in 2008.


1 - 78 S

36.

Test Bank for Intermediate Accounting, Fourteenth Edition

The occurrence that most likely would have no effect on 2012 net income is the a. sale in 2012 of an office building contributed by a stockholder in 1961. b. collection in 2012 of a dividend from an investment. c. correction of an error in the financial statements of a prior period discovered subsequent to their issuance. d. stock purchased in 1996 deemed worthless in 2012.

P

37. Which of the following is not a selling expense? a. Advertising expense b. Office salaries expense c. Freight-out d. Store supplies consumed

P

38.

The accountant for the Lintz Sales Company is preparing the income statement for 2012 and the balance sheet at December 31, 2012. The January 1, 2012 merchandise inventory balance will appear a. only as an asset on the balance sheet. b. only in the cost of goods sold section of the income statement. c. as a deduction in the cost of goods sold section of the income statement and as a current asset on the balance sheet. d. as an addition in the cost of goods sold section of the income statement and as a current asset on the balance sheet.

39.

In order to be classified as an extraordinary item in the income statement, an event or transaction should be a. unusual in nature, infrequent, and material in amount. b. unusual in nature and infrequent, but it need not be material. c. infrequent and material in amount, but it need not be unusual in nature. d. unusual in nature and material, but it need not be infrequent. Classification as an extraordinary item on the income statement would be appropriate for the a. gain or loss on disposal of a component of the business. b. substantial write-off of obsolete inventories. c. loss from a strike. d. none of these.

40.

41.

Which of these is generally an example of an extraordinary item? a. Loss incurred because of a strike by employees. b. Write-off of deferred marketing costs believed to have no future benefit. c. Gain resulting from the devaluation of the U.S. dollar. d. Gain resulting from the state exercising its right of eminent domain on a piece of land used as a parking lot.

42.

Under which of the following conditions would material flood damage be considered an extraordinary item for financial reporting purposes? a. Only if floods in the geographical area are unusual in nature and occur infrequently.


Financial Accounting and Accounting Standards

1 - 79

b. Only if the flood damage is material in amount and could have been reduced by prudent management. c. Under any circumstances as an extraordinary item. d. Flood damage should never be classified as an extraordinary item. 43.

An item that should be classified as an extraordinary item is a. write-off of goodwill. b. gains from transactions involving foreign currencies. c. losses from moving a plant to another city. d. gains from a company selling the only investment it has ever owned.

44.

How should an unusual event not meeting the criteria for an extraordinary item be disclosed in the financial statements? a. Shown as a separate item in operating revenues or expenses if material and supplemented by a footnote if deemed appropriate. b. Shown in operating revenues or expenses if material but not shown as a separate item. c. Shown net of income tax after ordinary net earnings but before extraordinary items. d. Shown net of income tax after extraordinary items but before net earnings.

45.

Which of the following is a change in accounting principle? a. A change in the estimated service life of machinery b. A change from FIFO to LIFO c. A change from straight-line to double-declining-balance d. A change from FIFO to LIFO and a change from straight-line to double-decliningbalance

46.

Which of the following is never classified as an extraordinary item? a. Losses from a major casualty. b. Losses from an expropriation of assets. c. Gain on a sale of the only security investment a company has ever owned. d. Losses from exchange or translation of foreign currencies.

47.

Which of the following is a required disclosure in the income statement when reporting the disposal of a component of the business? a. The gain or loss on disposal should be reported as an extraordinary item. b. Results of operations of a discontinued component should be disclosed immediately below extraordinary items. c. Earnings per share from both continuing operations and net income should be disclosed on the face of the income statement. d. The gain or loss on disposal should not be segregated, but should be reported together with the results of continuing operations.

48.

When a company discontinues an operation and disposes of the discontinued operation (component), the transaction should be included in the income statement as a gain or loss on disposal reported as a. a prior period adjustment. b. an extraordinary item.


Test Bank for Intermediate Accounting, Fourteenth Edition

1 - 80

c. an amount after continuing operations and before extraordinary items. d. a bulk sale of plant assets included in income from continuing operations. S

49.

A material item which is unusual in nature or infrequent in occurrence, but not both should be shown in the income statement a. b. c. d.

Net of Tax No No Yes Yes No Yes Yes No

Disclosed Separately

50.

Income taxes are allocated to a. extraordinary items. b. discontinued operations. c. prior period adjustments. d. all of these.

51.

Which of the following is true about intraperiod tax allocation? a. It arises because certain revenue and expense items appear in the income statement either before or after they are included in the tax return. b. It is required for extraordinary items and cumulative effect of accounting changes but not for prior period adjustments. c. Its purpose is to allocate income tax expense evenly over a number of accounting periods. d. Its purpose is to relate the income tax expense to the items which affect the amount of tax.

52.

Companies use intraperiod tax allocation for all of the following items except a. Discontinued operations. b. Extraordinary items. c. Changes in accounting estimates. d. Income from continuing operations.


Financial Accounting and Accounting Standards

1 - 81

53.

Which of the following items would be reported net of tax on the face of the income statement? a. Prior period adjustment b. Unusual gain c. Cumulative effect of a change in an accounting principle d. Discontinued operations

54.

Which of the following items would be reported at its gross amount on the face of the income statement? a. Extraordinary loss b. Prior period adjustment c. Cumulative effect of a change in an accounting principle d. Unusual gain

55.

Where must earnings per share be disclosed in the financial statements to satisfy generally accepted accounting principles? a. On the face of the statement of retained earnings (or, statement of stockholders' equity.) b. In the footnotes to the financial statements. c. On the face of the income statement. d. Either (a) or (c).

56.

Which of the following earnings per share figures must be disclosed on the face of the income statement? a. EPS on income from continuing operations. b. The effect on EPS from operations of a discontinued division, net of taxes. c. The effect on EPS from an extraordinary item, net of taxes. d. All of the above.

57.

Which of the following earnings per share figures must be disclosed on the face of the income statement? a. EPS for income before taxes. b. The effect on EPS from unusual items. c. EPS for gross profit. d. EPS for income from continuing operations.

S

Earnings per share should always be shown separately for a. net income and gross margin. b. net income and pretax income. c. income before extraordinary items. d. extraordinary items and prior period adjustments.

P

A correction of an error in prior periods' income will be reported

58.

59.

a. b. c. d.

In the income statement Yes Yes No No Yes No No Yes

Net of tax


1 - 82

Test Bank for Intermediate Accounting, Fourteenth Edition


Financial Accounting and Accounting Standards

1 - 83

60.

Which of the following items will not appear in the retained earnings statement? a. Net loss b. Prior period adjustment c. Discontinued operations d. Dividends

61.

Which one of the following types of losses is excluded from the determination of net income in income statements? a. Material losses resulting from transactions in the company's investments account. b. Material losses resulting from unusual sales of assets not acquired for resale. c. Material losses resulting from the write-off of intangibles. d. Material losses resulting from correction of errors related to prior periods.

62.

Watts Corporation made a very large arithmetical error in the preparation of its year-end financial statements by improper placement of a decimal point in the calculation of depreciation. The error caused the net income to be reported at almost double the proper amount. Correction of the error when discovered in the next year should be treated as a. an increase in depreciation expense for the year in which the error is discovered. b. a component of income for the year in which the error is discovered, but separately listed on the income statement and fully explained in a note to the financial statements. c. an extraordinary item for the year in which the error was made. d. a prior period adjustment.

63.

A company is not required to report a per share amount on the face of the income statement for which of the following items? a. Net income b. Prior period adjustment c. Extraordinary item d. Discontinued operations

64.

Earnings per share data are required on the face of which of the following financial statements? a. Statement of retained earnings b. Statement of stockholders' equity c. Income statement d. Balance sheet

65.

Which of the following is included in comprehensive income? a. Investments by owners. b. Unrealized gains on available-for-sale securities. c. Distributions to owners. d. Changes in accounting principles.

66.

Which of the following is not an acceptable way of displaying the components of other comprehensive income? a. Combined statement of retained earnings b. Second income statement


1 - 84

Test Bank for Intermediate Accounting, Fourteenth Edition

c. Combined statement of comprehensive income d. As part of the statement of stockholders' equity


Financial Accounting and Accounting Standards

1 - 85

67.

Which disclosure method do most companies use to display the components of other comprehensive income? a. Combined statement of retained earnings b. Second income statement c. Combined statement of comprehensive income d. As part of the statement of stockholders' equity

68.

Comprehensive income includes all of the following except a. dividend revenue. b. losses on disposal of assets. c. investments by owners. d. unrealized holding gains.

69.

The approach most companies use to provide information related to the components of other comprehensive income is a a. second separate income statement. b. combined income statement of comprehensive income. c. separate column in the statement of changes in stockholders’ equity. d. footnote disclosure.

Multiple Choice Answers—Conceptual Solution to Multiple Choice question for which the answer is “none of these.” 40. Many answers are possible.


1 - 86

Test Bank for Intermediate Accounting, Fourteenth Edition

MULTIPLE CHOICE—Computational 70.

Ortiz Co. had the following account balances: Sales revenue $ 180,000 Cost of goods sold 90,000 Salaries and wages expense 15,000 Depreciation expense 30,000 Dividend revenue 6,000 Utilities expense 12,000 Rent revenue 30,000 Interest expense 18,000 Sales returns and allow. 16,500 Advertising expense 19,500 What would Ortiz report as total revenues in a single-step income statement? a. $199,500 b. $ 15,000 c. $216,000 d. $180,000

71.

Ortiz Co. had the following account balances: Sales revenue $ 180,000 Cost of goods sold 90,000 Salaries and wages expense 15,000 Depreciation expense 30,000 Dividend revenue 6,000 Utilities expense 12,000 Rent revenue 30,000 Interest expense 18,000 Sales returns and allow. 16,500 Advertising expense 19,500 What would Ortiz report as total expenses in a single-step income statement? a. $190,500 b. $201,000 c. $184,500 d. $ 94,500

72.

For Mortenson Company, the following information is available: Cost of goods sold Dividend revenue Income tax expense Operating expenses Sales revenue

$120,000 5,000 12,000 46,000 200,000

In Mortenson’s single-step income statement, gross profit a. should not be reported. b. should be reported at $27,000.


Financial Accounting and Accounting Standards

c. should be reported at $80,000. d. should be reported at $85,000.

1 - 87


Test Bank for Intermediate Accounting, Fourteenth Edition

1 - 88

73.

For Mortenson Company, the following information is available: Cost of goods sold Dividend revenue Income tax expense Operating expenses Sales revenue

$120,000 5,000 12,000 46,000 200,000

In Mortenson’s multiple-step income statement, gross profit a. should not be reported b. should be reported at $27,000. c. should be reported at $80,000. d. should be reported at $85,000. 74.

For Rondelli Company, the following information is available: Cost of goods sold Dividend revenue Income tax expense Operating expenses Sales revenue a. b. c. d.

$270,000 12,000 27,000 105,000 450,000

In Rondelli's multiple-step income statement, gross profit should not be reported should be reported at $60,000. should be reported at $180,000. should be reported at $192,000.

75.

Gross billings for merchandise sold by Lang Company to its customers last year amounted to $12,720,000; sales returns and allowances were $370,000, sales discounts were $175,000, and freight-out was $140,000. Net sales last year for Lang Company were a. $12,720,000. b. $12,350,000. c. $12,175,000. d. $12,035,000.

76.

If plant assets of a manufacturing company are sold at a gain of $1,640,000 less related taxes of $500,000, and the gain is not considered unusual or infrequent, the income statement for the period would disclose these effects as a. a gain of $1,640,000 and an increase in income tax expense of $500,000. b. operating income net of applicable taxes, $1,140,000. c. a prior period adjustment net of applicable taxes, $1,140,000. d. an extraordinary item net of applicable taxes, $1,140,000.

77.

Manning Company has the following items: write-down of inventories, $360,000; loss on disposal of Sports Division, $555,000; and loss due to strike, $339,000. Ignoring income taxes, what total amount should Manning Company report as extraordinary losses? a. $ -0-. b. $555,000.


Financial Accounting and Accounting Standards

c. $699,000. d. $894,000.

1 - 89


1 - 90

Test Bank for Intermediate Accounting, Fourteenth Edition

78.

Garwood Company has the following items: write-down of inventories, $360,000; loss on disposal of Sports Division, $555,000; and loss due to an expropriation, $339,000. Ignoring income taxes, what total amount should Garwood Company report as extraordinary losses? a. $339,000 b. $555,000. c. $699,000. d. $894,000.

79.

An income statement shows “income before income taxes and extraordinary items” in the amount of $2,740,000. The income taxes payable for the year are $1,440,000, including $480,000 that is applicable to an extraordinary gain. Thus, the “income before extraordinary items” is a. $1,780,000. b. $820,000. c. $1,860,000. d. $900,000.

80.

Dole Company, with an applicable income tax rate of 30%, reported net income of $350,000. Included in income for the period was an extraordinary loss from flood damage of $50,000 before deducting the related tax effect. The company's income before income taxes and extraordinary items was a. $400,000. b. $500,000. c. $550,000. d. $385,000.

81.

A review of the December 31, 2012, financial statements of Somer Corporation revealed that under the caption "extraordinary losses," Somer reported a total of $1,030,000. Further analysis revealed that the $1,030,000 in losses was comprised of the following items: (1) Somer recorded a loss of $300,000 incurred in the abandonment of equipment formerly used in the business. (2) In an unusual and infrequent occurrence, a loss of $500,000 was sustained as a result of hurricane damage to a warehouse. (3) During 2012, several factories were shut down during a major strike by employees, resulting in a loss of $170,000. (4) Uncollectible accounts receivable of $60,000 were written off as uncollectible. Ignoring income taxes, what amount of loss should Somer report as extraordinary on its 2012 income statement? a. $300,000. b. $500,000. c. $800,000. d. $1,030,000.


Financial Accounting and Accounting Standards

1 - 91

82. At Ruth Company, events and transactions during 2012 included the following. The tax rate for all items is 30%. (1) Depreciation for 2010 was found to be understated by $60,000. (2) A strike by the employees of a supplier resulted in a loss of $50,000. (3) The inventory at December 31, 2010 was overstated by $80,000. (4) A flood destroyed a building that had a book value of $1,000,000. Floods are very uncommon in that area. The effect of these events and transactions on 2012 income from continuing operations net of tax would be a. ($35,000). b. ($77,000). c. ($133,000). d. ($833,000). 83. At Ruth Company, events and transactions during 2012 included the following. The tax rate for all items is 30%. (1) Depreciation for 2010 was found to be understated by $60,000. (2) A strike by the employees of a supplier resulted in a loss of $50,000. (3) The inventory at December 31, 2010 was overstated by $80,000. (4) A flood destroyed a building that had a book value of $1,000,000. Floods are very uncommon in that area. The effect of these events and transactions on 2012 net income net of tax would be a. ($35,000). b. ($735,000). c. ($777,000). d. ($833,000). 84.

During 2012, Lopez Corporation disposed of Pine Division, a major component of its business. Lopez realized a gain of $1,800,000, net of taxes, on the sale of Pine's assets. Pine's operating losses, net of taxes, were $2,100,000 in 2012. How should these facts be reported in Lopez's income statement for 2012? Total Amount to be Included in Income from Results of Continuing Operations Discontinued Operations a. $2,100,000 loss$1,800,000 gain b. 300,000 loss 0 c. 0300,000 loss d. 1,800,000 gain2,100,000 loss

85.

Sandstrom Corporation has an extraordinary loss of $150,000, an unusual gain of $105,000, and a tax rate of 40%. At what amount should Sandstrom report each item? Extraordinary loss Unusual gain a. $(150,000) $105,000 b. (150,000) 63,000


Test Bank for Intermediate Accounting, Fourteenth Edition

1 - 92

c. d.

(90,000) (90,000)

105,000 63,000

86.

Prophet Corporation has an extraordinary loss of $600,000, an unusual gain of $420,000, and a tax rate of 40%. At what amount should Prophet report each item? Extraordinary loss Unusual gain a. $(600,000) $420,000 b. (600,000) 252,000 c. (360,000) 420,000 d. (360,000) 252,000

87.

Arreaga Corp. has a tax rate of 40 percent and income before non-operating items of $464,000. It also has the following items (gross amounts). Unusual loss Extraordinary loss Gain on disposal of equipment Change in accounting principle increasing prior year's income

$ 74,000 202,000 16,000 106,000

What is the amount of income tax expense Arreaga would report on its income statement? a. $185,600 b. $162,400 c. $198,400 d. $124,000 88.

Palomo Corp has a tax rate of 30 percent and income before non-operating items of $714,000. It also has the following items (gross amounts). Unusual gain Loss from discontinued operations Dividend revenue Income increasing prior period adjustment

$ 46,000 366,000 12,000 148,000

What is the amount of income tax expense Palomo would report on its income statement? a. $231,600 b. $121,800 c. $166,200 d. $217,800 89.

Lantos Company had a 40 percent tax rate. Given the following pre-tax amounts, what would be the income tax expense reported on the face of the income statement? Sales revenue $ 300,000 Cost of goods sold 180,000 Salaries and wages expense 24,000 Depreciation expense 33,000 Dividend revenue 27,000 Utilities expense 3,000


Financial Accounting and Accounting Standards

a. b. c. d.

Extraordinary loss Interest expense $32,400 $20,400 $21,600 $ 9,600

30,000 6,000

1 - 93


Test Bank for Intermediate Accounting, Fourteenth Edition

1 - 94

90.

In 2012, Esther Corporation reported net income of $600,000. It declared and paid preferred stock dividends of $150,000 and common stock dividends of $60,000. During 2012, Esther had a weighted average of 200,000 common shares outstanding. Compute Esther's 2012 earnings per share. a. b. c. d.

91.

In 2012, Linz Corporation reported an extraordinary loss of $1,000,000, net of tax. It declared and paid preferred stock dividends of $100,000 and common stock dividends of $300,000. During 2012, Linz had a weighted average of 400,000 common shares outstanding. Compute the effect of the extraordinary loss, net of tax, on earnings per share. a. b. c. d.

92.

$1.50 $1.75 $2.25 $2.50

In 2012, Benfer Corporation reported net income of $280,000. It declared and paid common stock dividends of $32,000 and had a weighted average of 70,000 common shares outstanding. Compute the earnings per share to the nearest cent. a. b. c. d.

93.

$1.95 $2.25 $3.00 $3.75

$3.54 $2.80 $3.60 $4.00

Benedict Corporation reports the following information: Net income Dividends on common stock Dividends on preferred stock Weighted average common shares outstanding

$750,000 210,000 90,000 100,000

Benedict should report earnings per share of a. $4.50. b. $5.40 c. $6.60. d. $7.50. 94.

Norling Corporation reports the following information: Net income Dividends on common stock Dividends on preferred stock Weighted average common shares outstanding Norling should report earnings per share of a. $2.25.

$750,000 210,000 90,000 200,000


Financial Accounting and Accounting Standards

b. $2.70 c. $3.30. d. $3.75. 95.

Moorman Corporation reports the following information: Correction of understatement of depreciation expense in prior years, net of tax $ 645,000 Dividends declared 480,000 Net income 1,500,000 Retained earnings, 1/1/12, as reported 3,000,000 Moorman should report retained earnings, 1/1/12, as adjusted at a. $2,355,000. b. $3,000,000. c. $3,645,000. d. $4,665,000.

96.

Moorman Corporation reports the following information: Correction of understatement of depreciation expense in prior years, net of tax $ 645,000 Dividends declared 480,000 Net income 1,500,000 Retained earnings, 1/1/12, as reported 3,000,000 Moorman should report retained earnings, 12/31/12, as adjusted at a. $2,355,000. b. $3,375,000. c. $4,020,000. d. $4,665,000.

97.

Leonard Corporation reports the following information: Correction of overstatement of depreciation expense in prior years, net of tax $ 215,000 Dividends declared 160,000 Net income 500,000 Retained earnings, 1/1/12, as reported 2,000,000 Leonard should report retained earnings, 1/1/12, as adjusted at a. $1,785,000. b. $2,000,000. c. $2,215,000. d. $2,555,000.

98.

Leonard Corporation reports the following information: Correction of overstatement of depreciation expense in prior years, net of tax $ 215,000 Dividends declared 160,000

1 - 95


1 - 96

Test Bank for Intermediate Accounting, Fourteenth Edition

Net income Retained earnings, 1/1/12, as reported

99.

500,000 2,000,000

Leonard should report retained earnings, 12/31/12, at a. $1,785,000. b. $2,125,000. c. $2,340,000. d. $2,555,000. The following information was extracted from the accounts of Essex Corporation at December 31, 2012: CR(DR) Total reported income since incorporation $3,400,000 Total cash dividends paid (1,600,000) Unrealized holding loss (240,000) Total stock dividends distributed (400,000) Prior period adjustment, recorded January 1, 2012 150,000 What should be the balance of retained earnings at December 31, 2012? a. $1,310,000. b. $1,400,000. c. $1,160,000. d. $1,550,000.

100.

Madsen Company reported the following information for 2012: Sales revenue $1,530,000 Cost of goods sold 1,050,000 Operating expenses 165,000 Unrealized holding gain on available-for-sale securities 120,000 Cash dividends received on the securities 6,000 For 2012, Madsen would report other comprehensive income of a. $411,000. b. $405,000. c. $126,000. d. $120,000.

101.

Korte Company reported the following information for 2012: Sales revenue Cost of goods sold Operating expenses Unrealized holding gain on available-for-sale securities Cash dividends received on the securities For 2012, Korte would report comprehensive income of a. $351,000. b. $345,000. c. $291,000. d. $60,000.

$1,500,000 1,050,000 165,000 60,000 6,000


Financial Accounting and Accounting Standards

102.

For the year ended December 31, 2012, Transformers Inc. reported the following: Net income Preferred dividends declared Common dividend declared Unrealized holding loss, net of tax Retained earnings Common stock Accumulated Other Comprehensive Income, Beginning Balance

$120,000 20,000 4,000 2,000 160,000 80,000 10,000

What would Transformers report as its ending balance of Accumulated Other Comprehensive Income? a. b. c. d. 103.

$12,000 $10,000 $8,000 $2,000

For the year ended December 31, 2012, Transformers Inc. reported the following: Net income $120,000 Preferred dividends declared 20,000 Common dividend declared 4,000 Unrealized holding loss, net of tax 2,000 Retained earnings, beginning balance 160,000 Common stock 80,000 Accumulated Other Comprehensive Income, Beginning Balance 10,000 What would Transformers report as the ending balance of Retained Earnings? a. b. c. d.

104.

$278,000 $266,000 $256,000 $254,000

For the year ended December 31, 2012, Transformers Inc. reported the following: Net income $120,000 Preferred dividends declared 20,000 Common dividend declared 4,000 Unrealized holding loss, net of tax 2,000 Retained earnings, beginning balance 160,000 Common stock 80,000 Accumulated Other Comprehensive Income, Beginning Balance 10,000 What would Transformers report as total stockholders' equity? a. $344,000 b. $336,000 c. $256,000

1 - 97


1 - 98

Test Bank for Intermediate Accounting, Fourteenth Edition

d. $240,000


Financial Accounting and Accounting Standards

1 - 99

Multiple Choice Answers—Computational

MULTIPLE CHOICE—CPA Adapted 105. Perry Corp. reports operating expenses in two categories: (1) selling and (2) general and administrative. The adjusted trial balance at December 31, 2012, included the following expense accounts: Accounting and legal fees Advertising Freight-out Interest Loss on sale of long-term investments Officers' salaries Rent for office space Sales salaries and commissions

$280,000 240,000 150,000 120,000 60,000 360,000 360,000 220,000

One-half of the rented premises is occupied by the sales department. How much of the expenses listed above should be included in Perry's selling expenses for 2012? a. $460,000. b. $610,000. c. $640,000. d. $790,000. 106. Perry Corp. reports operating expenses in two categories: (1) selling and (2) general and administrative. The adjusted trial balance at December 31, 2012, included the following expense accounts: Accounting and legal fees Advertising Freight-out Interest Loss on sale of long-term investments Officers' salaries Rent for office space Sales salaries and commissions

$280,000 240,000 150,000 120,000 60,000 360,000 360,000 220,000

One-half of the rented premises is occupied by the sales department. How much of the expenses listed above should be included in Perry's general and administrative expenses for 2012? a. $820,000. b. $880,000. c. $940,000.


1 - 100 Test Bank for Intermediate Accounting, Fourteenth Edition

d. $1,000,000. 107.

Didde Corp. reports operating expenses in two categories: (1) selling and (2) general and administrative. The adjusted trial balance at December 31, 2012 included the following expense and loss accounts: Accounting and legal fees Advertising Freight-out Interest Loss on sale of long-term investment Officers' salaries Rent for office space Sales salaries and commissions

$210,000 270,000 120,000 105,000 45,000 335,000 330,000 255,000

One-half of the rented premises is occupied by the sales department. Didde's total selling expenses for 2012 are a. $810,000. b. $690,000. c. $645,000. d. $555,000. 108.

The following items were among those that were reported on Dye Co.'s income statement for the year ended December 31, 2012: Legal and audit fees $390,000 Rent for office space 540,000 Interest on inventory floor plan 630,000 Loss on abandoned equipment used in operations 105,000 The office space is used equally by Dye's sales and accounting departments. What amount of the above-listed items should be classified as general and administrative expenses in Dye's multiple-step income statement? a. $660,000. b. $765,000. c. $930,000. d. $1,290,000.

109. Logan Corp.'s trial balance of income statement accounts for the year ended December 31, 2012 included the following: Debit Credit Sales revenue $280,000 Cost of goods sold $100,000 Administrative expenses 50,000 Loss on disposal of equipment 18,000 Sales commission expense 16,000 Interest revenue 10,000 Freight-out 6,000 Loss due to earthquake damage 24,000


Financial Accounting and Accounting Standards

Bad debt expense Totals

6,000 $220,000

1 - 101

$290,000

Other information: Logan's income tax rate is 30%. Finished goods inventory: January 1, 2012 $160,000 December 31, 2012 140,000 On Logan's multiple-step income statement for 2012, Cost of goods manufactured is a. $126,000. b. $120,000. c. $86,000. d. $80,000. 110. Logan Corp.'s trial balance of income statement accounts for the year ended December 31, 2012 included the following: Debit Credit Sales revenue $280,000 Cost of good sold $100,000 Administrative expenses 50,000 Loss on disposal of equipment 18,000 Sales commission expense 16,000 Interest revenue 10,000 Freight-out 6,000 Loss due to earthquake damage 24,000 Bad debt expense 6,000 Totals $220,000 $290,000 Other information: Logan's income tax rate is 30%. Finished goods inventory: January 1, 2012 $160,000 December 31, 2012 140,000 On Logan's multiple-step income statement for 2012, Income before extraordinary item is a. $128,000. b. $94,000. c. $65,800. d. $49,000.


1 - 102 Test Bank for Intermediate Accounting, Fourteenth Edition

111. Logan Corp.'s trial balance of income statement accounts for the year ended December 31, 2012 included the following: Debit Credit Sales $280,000 Cost of sales $100,000 Administrative expenses 50,000 Loss on sale of equipment 18,000 Commissions to salespersons 16,000 Interest revenue 10,000 Freight-out 6,000 Loss due to earthquake damage 24,000 Bad debt expense 6,000 Totals $220,000 $290,000 Other information: Logan's income tax rate is 30%. Finished goods inventory: January 1, 2012 $160,000 December 31, 2012 140,000 On Logan's multiple-step income statement for 2012, Extraordinary loss is a. $16,800. b. $24,000. c. $29,400. d. $42,000. 112.

Chase Corp. had the following infrequent transactions during 2012: A $225,000 gain from selling the only investment Chase has ever owned. A $315,000 gain on the sale of equipment. A $105,000 loss on the write-down of inventories. In its 2012 income statement, what amount should Chase report as total infrequent net gains that are not considered extraordinary? a. $120,000. b. $210,000. c. $435,000. d. $540,000.

113.

James, Inc. incurred the following infrequent losses during 2012: A $140,000 write-down of equipment leased to others. A $80,000 adjustment of accruals on long-term contracts. A $120,000 write-off of obsolete inventory. In its 2012 income statement, what amount should James report as total infrequent losses that are not considered extraordinary? a. $340,000.


Financial Accounting and Accounting Standards

b. $260,000. c. $220,000. d. $200,000.

1 - 103


1 - 104 Test Bank for Intermediate Accounting, Fourteenth Edition

114.

Which of the following should be reported as a prior period adjustment?

a. b. c. d.

Change in Estimated Lives Change from Unaccepted of Depreciable Assets Principle to Accepted Principle Yes Yes No Yes Yes No No No

Multiple Choice Answers—CPA Adapted

IFRS QUESTIONS True/False 1. Both U.S. GAAP and IFRS discuss income statement presentation using either a single-step or multi-step approach. 2. IFRS does not allow gains or losses to be classified as extraordinary items. 3. IFRS allows for revaluation of long-term tangible and intangible assets with the differences impacting equity but not net income. 4. Both IFRS and U.S. GAAP allow for comprehensive income to be reported in either a Statement of Stockholders' Equity or a Statement of Recognized Income and Expense. 5. Under IFRS, a company may classify expenses by function, but must also disclose the classification of expenses by nature. Answers to True/False: Multiple Choice: 1. The IFRS income statement classification of expenses by nature results in descriptions which include all of the following except a. salaries b. depreciation c. distribution d. utilities 2. U.S. GAAP allows all of the following statement formats to be used for reporting comprehensive income except a. Statement of Recognized Income and Expense b. Single Income Statement c. Combined Income Statement of Comprehensive Income d. Statement of Stockholders' Equity 3. An IFRS SoRIE statement might include all of the following except a. net income or loss b. unrealized gains or losses on the revaluation of long-term assets


Financial Accounting and Accounting Standards

c. cumulative effect of a change in accounting principle d. extraordinary gain or loss Answers to Multiple Choice:

1 - 105


1 - 106 Test Bank for Intermediate Accounting, Fourteenth Edition

Short Answer: 1. What are the IFRS requirements with respect to expense classification? 2. Bradshaw Company experienced a loss that was deemed to be both unusual in nature and infrequent in occurrence. How should Bradshaw report this item in accordance with IFRS?


Turn static files into dynamic content formats.

Create a flipbook
Issuu converts static files into: digital portfolios, online yearbooks, online catalogs, digital photo albums and more. Sign up and create your flipbook.